neurology answers

43
Item 1 Answer: D Normal electrolyte values in a pregnant woman Educational Objective: Understand the normal acid-base physiologic response to pregnancy. Normal pregnancy causes progressive respiratory alalosis! and by the third trimester the average arterial blood "co# has $allen to about %& mm 'g. (his causes an appropriate compensatory reduction in bicarbonate to about #& me)*+. (here$ore! this patient,s serum chemistry results are entirely appropriate and normal $or a woman in the third trimester o$ pregnancy. (here is nothing to suggest any $orm o$ metabolic acidosis. Although she could have respiratory alalosis! this diagnosis cannot be d etermined based on the serum chemistry results provided. ibliography .+im /0! 1at2 A3! +indheimer 4D. Acid-base regulation in pregnancy. Am 5 "hysiol. 6789#%:78-6. #.+andon 4. Acid-base disorders dur ing pregnancy. ;lin Obstet <ynecol. 669%7:8-# Item 2 Answer: A Discontinue enalapril9 measure serum potassium and creatinine levels Educational Objective : =ecogni2e that angiotens in-converting en2yme inhibitors can cause acute renal $ailure. "atients with pree>isting chronic renal insu$$iciency should be treated to reduce the progression o$ renal disease. ;ontrol o$ hypertension by any regimen is e$$ective! but angiotensin-converting en2yme ?A;E@ inhibitors have been shown to con$er enhanced bene$its in several studies o$ patients with and without diabetic renal disease. (his patient most liely has chronic renal insu$$iciency! as evidenced by an increased serum creatinine concentration. (he cause is unclear! but hypertensive nephropathy is a possible diagnosis! especially because o$ the relatively low level o$ urinary protein e>cretion. (he patient also has a history o$ hypercholesterolemia and evidence o$ peripheral vascular disease! raising the possibility o$ the presence o$ renal arterial disease. A d iagnosis o$ acute renal $ailure can be con$i rmed in this patient! who already has chronic renal $ailure! because his serum creatinine concentration has increased over a short period. "atients with renal vascular disease or pree>isting renal insu$$iciency! as well as those with decreased renal per$usion because o$ congestive heart $ailure! may develop an acute decrease in the glomerular $iltration rate when treated with A;E inhibitors ?or angiotensin 33 receptor blocers@ because o$ the hemodynamic e$$ects o$ these drugs. "hysical e>amination demonstrates that this patient does not have a volume disorder! and his urinalysis does not suggest glomerular! interstitial! or renal tubular disease. Although the diagnosis is liely to be renal vascular disease! the decrement in the glomerular $ iltration rate associated with blocade o$ angiotensin synthesis is usually )uicly reversed when the A;E inhibitor is discontinued. +ovastatin has been associated with the development o$ rhabdo myolysis! but there is no evidence o$ hemoglobin or myoglobin in this patient,s urine or symptoms o$ muscle pain or weaness. (here is also no evidence o$ volume depletion and there$ore no indication $or administering intravenous $luids. ;ontrast agents should be used only $or the most urgent indications in patients with acute renal $ailure because o$ the potential $or the development o$ nephroto>icity. inally ! there is no evidence that the b-blocer! given in an unchanged dose! has caused this patient,s renal $unction to deteriorate. ibliography .iesenbach <! 5ano O! 0tuby U! Bagorni 5. 4yoglobinuric renal $ailure due to long-standing lovastatin therapy in a patient with pre-e>isting chronic renal insu$$iciency. Nephrol Dial (r ansplant. 6689:#&C6- 8&. #.1alra "A! 1umwenda 4! 4acDowall "! =oland 4O. uestionnaire study and audit o$ use o $ angiotensin-converting en2yme inhibitor and monitoring in general practice: the need $or guidelines to prevent renal $ailure. 45. 6669%:#%-7. %.Esmail BN! +oewen "0. +osartan as an alternative to A;E inhibitors in patients with renal dys$unction. Ann "harmacother . 669%#:&68-. .Fyncel A! Ebiili ! 4elin 5"! =a ndou> ;! +avaud 0! ;hanard 5. +ong-term $ollow-up o$ acute renal $ailure caused by angiotensin-converting en2yme inhibitors. Am 5 'ypertens. 669:&&-8.

Upload: benjamin-nelson

Post on 03-Jun-2018

222 views

Category:

Documents


0 download

TRANSCRIPT

Page 1: Neurology Answers

8/11/2019 Neurology Answers

http://slidepdf.com/reader/full/neurology-answers 1/43

Item 1

Answer: DNormal electrolyte values in a pregnant womanEducational Objective: Understand the normal acid-base physiologic response to pregnancy.Normal pregnancy causes progressive respiratory alalosis! and by the third trimester theaverage arterial blood "co# has $allen to about %& mm 'g. (his causes an appropriatecompensatory reduction in bicarbonate to about #& me)*+. (here$ore! this patient,s serum

chemistry results are entirely appropriate and normal $or a woman in the third trimester o$pregnancy. (here is nothing to suggest any $orm o$ metabolic acidosis. Although she couldhave respiratory alalosis! this diagnosis cannot be determined based on the serum chemistryresults provided.ibliography.+im /0! 1at2 A3! +indheimer 4D. Acid-base regulation in pregnancy. Am 5 "hysiol.6789#%:78-6.#.+andon 4. Acid-base disorders during pregnancy. ;lin Obstet <ynecol. 669%7:8-#

Item 2

Answer: ADiscontinue enalapril9 measure serum potassium and creatinine levelsEducational Objective: =ecogni2e that angiotensin-converting en2yme inhibitors can causeacute renal $ailure.

"atients with pree>isting chronic renal insu$$iciency should be treated to reduce theprogression o$ renal disease. ;ontrol o$ hypertension by any regimen is e$$ective! butangiotensin-converting en2yme ?A;E@ inhibitors have been shown to con$er enhanced bene$itsin several studies o$ patients with and without diabetic renal disease. (his patient most lielyhas chronic renal insu$$iciency! as evidenced by an increased serum creatinine concentration.(he cause is unclear! but hypertensive nephropathy is a possible diagnosis! especially becauseo$ the relatively low level o$ urinary protein e>cretion. (he patient also has a history o$hypercholesterolemia and evidence o$ peripheral vascular disease! raising the possibility o$ thepresence o$ renal arterial disease. A diagnosis o$ acute renal $ailure can be con$irmed in thispatient! who already has chronic renal $ailure! because his serum creatinine concentration hasincreased over a short period. "atients with renal vascular disease or pree>isting renalinsu$$iciency! as well as those with decreased renal per$usion because o$ congestive heart$ailure! may develop an acute decrease in the glomerular $iltration rate when treated with A;Einhibitors ?or angiotensin 33 receptor blocers@ because o$ the hemodynamic e$$ects o$ these

drugs. "hysical e>amination demonstrates that this patient does not have a volume disorder!and his urinalysis does not suggest glomerular! interstitial! or renal tubular disease. Althoughthe diagnosis is liely to be renal vascular disease! the decrement in the glomerular $iltrationrate associated with blocade o$ angiotensin synthesis is usually )uicly reversed when theA;E inhibitor is discontinued.+ovastatin has been associated with the development o$ rhabdomyolysis! but there is noevidence o$ hemoglobin or myoglobin in this patient,s urine or symptoms o$ muscle pain orweaness. (here is also no evidence o$ volume depletion and there$ore no indication $oradministering intravenous $luids. ;ontrast agents should be used only $or the most urgentindications in patients with acute renal $ailure because o$ the potential $or the development o$nephroto>icity. inally! there is no evidence that the b-blocer! given in an unchanged dose!has caused this patient,s renal $unction to deteriorate.ibliography.iesenbach <! 5ano O! 0tuby U! Bagorni 5. 4yoglobinuric renal $ailure due to long-standing

lovastatin therapy in a patient with pre-e>isting chronic renal insu$$iciency. Nephrol Dial(ransplant. 6689:#&C6-8&.#.1alra "A! 1umwenda 4! 4acDowall "! =oland 4O. uestionnaire study and audit o$ use o$angiotensin-converting en2yme inhibitor and monitoring in general practice: the need $orguidelines to prevent renal $ailure. 45. 6669%:#%-7.%.Esmail BN! +oewen "0. +osartan as an alternative to A;E inhibitors in patients with renaldys$unction. Ann "harmacother. 669%#:&68-..Fyncel A! Ebiili ! 4elin 5"! =andou> ;! +avaud 0! ;hanard 5. +ong-term $ollow-up o$ acuterenal $ailure caused by angiotensin-converting en2yme inhibitors. Am 5 'ypertens.669:&&-8.

Page 2: Neurology Answers

8/11/2019 Neurology Answers

http://slidepdf.com/reader/full/neurology-answers 2/43

C.van de /en "5! eutler 55! 1aatee =! ee 5! 4ali F"! 1oomans 'A. Angiotensin convertingen2yme inhibitor-induced renal dys$unction in atherosclerotic renovascular disease. 1idney 3nt.669C%:68-6%.

Item 3

Answer: (he serum creatinine value o$ .# mg*d+ represents a <= o$ less than C&G o$ normal

Educational Objective: 1now how to interpret the serum creatinine value as an estimate o$glomerular $iltration rate.(he baseline serum creatinine value o$ .# mg*d+ is de$initely abnormal! since in an elderlysmall woman with a small muscle mass! the serum creatinine value would be &.C mg*d+ to &.7mg*d+ i$ the glomerular $iltration rate ?<=@ was normal. Each doubling o$ the serumcreatinine value is e)uivalent to a C&G reduction o$ <=. 'ence! based on an e>pected normalserum creatinine value o$ &.C mg*d+ to &.7 mg*d+! the measured <= would be e>pected tobe less than C&G o$ normal. ;hanges in creatinine catabolism do not occur with age alone butcan occur during a severe illness. Although aging is sometimes accompanied by a decreased<=! this does not always happen. Fhen the <= does $all in an older person! it will bere$lected by a rising serum creatinine value. (he most important in$ormation when interpretinga border-line-normal serum creatinine level is the age and si2e o$ the patient ?which re$lectsthe patient,s muscle [email protected]

.Falser 4. Assessing renal $unction $rom creatinine measurements in adults with chronicrenal $ailure. Am 5 1idney Dis. 669 %#:#%-%.

Item 4

Answer: D=educed tubular secretion o$ creatinine due to trimethoprimEducational Objective: 1now how to interpret the serum creatinine value as an estimate o$glomerular $iltration rate.(he increase in serum creatinine $rom .# mg*d+ to . mg*d+ is most liely caused bydecreased tubular secretion o$ creatinine due to administration o$ trimethoprim. othcimetidine and trimethoprim reduce tubular secretion o$ creatinine and thus may cause smallbut signi$icant increases in the serum creatinine level in the absence o$ a reduction in theglomerular $iltration rate. Acute interstitial nephritis due to administration o$ trimethoprim-sul$ametho>a2ole is very rare. (he present $ormulations o$ sul$onamides do not precipitate in

the idney. inally! bladder outlet obstruction would be very unusual in a woman with thisclinical scenario.ibliography. Falser 4. Assessing renal $unction $rom creatinine measurements in adults with chronicrenal $ailure. Am 5 1idney Dis. 669 %#:#%-%.#. Andreev E! 1oopman 4! Aris2 +. A rise in plasma creatinine that is not a sign o$ renal$ailure: Fhich drugs can be responsibleH 5 3ntern 4ed. 6669#8:#7-C#.

Item 5

Answer: Doppler ultrasonography o$ the idneys and renal vasculatureEducational Objective: =ecogni2e the thromboembolic complications o$ the nephroticsyndrome.(his patient has the nephrotic syndrome with I urinary protein! edema! hypoalbuminemia!

and hyperlipidemia. Acute shortness o$ breath in an a$ebrile patient with hypo>emia!respiratory alalosis! and no in$iltrate on a chest radiograph all suggest a pulmonary embolismdespite the indeterminate ventilationJper$usion lung scan. 3n patients who are pro$oundlyhypoalbuminemic ?serum albumin K # g*d+@! pulmonary emboli $rom renal vein thrombosisshould be a leading consideration. (his is especially true in patients with idiopathicmembranous nephropathy! the most common pattern o$ idiopathic nephrotic syndrome inwhite patients. Doppler ultrasonography o$ the renal veins is one way to chec $or renal veinthrombosis. Alternative methods are ;( or nuclear magnetic venography. (his patient does nothave $indings o$ pericarditis or other cardiac lesions! which might be de$ined by anechocardiogram. 0erum total complement and complement ;% levels! although use$ul in

Page 3: Neurology Answers

8/11/2019 Neurology Answers

http://slidepdf.com/reader/full/neurology-answers 3/43

de$ining the cause o$ glomerular lesions in patients with the nephrotic syndrome! would not behelp$ul initially in this patient with acute dyspnea. +iewise! pulmonary $unction tests are notindicated in the acute setting.ibliography.Orth 0=! =it2 E. (he nephrotic syndrome. N Engl 5 4ed. 669%%:#&#-.#.=abelin (5! Bwaginga 55! 1oomans 'A! 0i>ma 55. (hrombosis and hemostasis in renaldisease. 1idney 3nt. 6698:#7-68.

%.Appel <. <lomerulonephritis. 3n: <oldman +! ennett 5;! eds. ;ecil (e>tboo o$ 4edicine.#st ed. "hiladelphia: F 0aunders9 #&&&:C8-6.

Item 6

Answer: ;0yndrome o$ inappropriate antidiuretic hormone secretion ?03AD'@Educational Objective: Understand the di$$erential diagnosis o$ hyponatremia and theimportance o$ the urinary sodium concentration! serum uric acid! blood urea nitrogen! andserum creatinine and the assessment o$ clinical volume status in establishing the diagnosis.(his patient has the syndrome o$ inappropriate antidiuretic hormone secretion ?03AD'@."atients with lung cancer! especially small cell lung cancer! $re)uently have 03AD'. (hesyndrome may be clinically )uiescent until the patient receives intravenous $luids $ollowingadmission to the hospital. Although this patient was given normal saline! his serum sodiumconcentration $ell because his idneys e>creted electrolytes at a concentration L C me)*+.

(here$ore! he was retaining electrolyte-$ree water. Although he has bilateral adrenal massesthat are liely due to metastatic disease! he is unliely to have Addison,s disease. 4ostpatients with adrenal metastatic disease do not develop Addison,s disease. urther-more! thelow potassium concentration argues against Addison,s disease. E>tracellular $luid volumedepletion is unliely! given the clinical scenario and the $act that his urinary sodium is &me)*+. Although he may have mild heart $ailure and hepatic disease! the clinical $indings!including the high urinary sodium concentration! mae cirrhosis and congestive heart $ailurevery unliely causes o$ the $all in the serum sodium concentration. (he serum uric acidconcentration also helps to di$$erentiate between these hyponatremic syndromes. Uric acidconcentration is usually elevated when the e>tracellular $luid volume is contracted or thee$$ective arterial volume is reduced in patients with cirrhosis or congestive heart $ailure. 3ncontrast! the uric acid concentration is usually reduced when the e>tracellular $luid volume ise>panded! as in patients with 03AD'.ibliography

.Decau> <! <enette ! 4ocel 5. 'ypouremia in the syndrome o$ inappropriate secretion o$antidiuretic hormone. Ann 3ntern 4ed. 6&96%: 78-7.#.0chrier =F. "athogenesis o$ sodium and water retention in high-output and low-outputcardiac $ailure! nephrotic syndrome! cirrhosis! and pregnancy ?#@. N Engl 5 4ed.69%6:#7-%.%.+eier ;/! Dei ;as +! 4etra 4. ;linical relevance and management o$ the major electrolyteabnormalities in congestive heart $ailure: hyponatremia! hypoalemia! and hypomagnesemia.Am 'eart 5. 669#:C8-7..Oster 5=! 0inger 3. 'yponatremia! hyposmolality! and hypotonicity: tables and $ables. Arch3ntern 4ed. 6669C6:%%%-8.

Item 7

Answer: ;=enal ultrasonography

Educational Objective: 3denti$y autosomal dominant polycystic idney disease as a secondary$orm o$ hypertension.(his patient! with stage # hypertension! a normal urinalysis and renal $unction! and a positive$amily history $or hypertension and renal disease! has autosomal dominant polycystic idneydisease ?AD"1D@. AD"1D may be $ound in young adults with hypertension! a2otemia! orpalpable idneys. 3n patients with this disorder! hypertension may occur even with a normalurinalysis and serum creatinine level. (he positive $amily history o$ renal $ailure is suggestiveo$ an inherited disorder. Establishing the diagnosis o$ AD"1D may be important $or prognosisand $or providing $amily counseling. =enal ultrasonography is indicated! as this study will showmultiple cysts in the idneys. Appro>imately C&G o$ patients may also have hepatic cysts.

Page 4: Neurology Answers

8/11/2019 Neurology Answers

http://slidepdf.com/reader/full/neurology-answers 4/43

Obtaining a captopril-stimulated renal scan and*or plasma renin activity and aldosteronedeterminations to search $or renal artery stenosis is not warranted. "atients with renal arterystenosis ?including $ibromuscular hyperplasia in young women@ or partial urinary tractobstruction may present with hypertension and a normal urinalysis and idney $unction.'owever! the positive $amily history o$ renal $ailure suggests the correct diagnosis o$ AD"1D. A#-hour urine determination $or vanillylmandelic acid is also not warranted because the patientdoes not have signs or symptoms suggestive o$ pheochromocytoma.

ibliography.<abow "A. Autosomal dominant polycystic idney disease. N Engl 5 4ed. 66%9%#6:%%#-#.#."ar$rey "0! ear 5;! 4organ 5! ;ramer ;! 4c4anamon "5! <ault 4'! et al. (he diagnosisand prognosis o$ autosomal dominant polycystic idney disease. N Engl 5 4ed.66&9%#%:&C-6&.%."errone =D. E>trarenal mani$estations o$ AD"1D. 1idney 3nt. 6679C:#&##-%8..=avine D! <ibson =N! Faler =<! 0he$$ield +5! 1incaid-0mith "! Dans D4. Evaluation o$ultrasonographic diagnostic criteria $or autosomal dominant polycystic idney disease .+ancet. 669%%:#-7.

Item 8

Answer: A=habdomyolysisEducational Objective: Evaluate acute renal $ailure in a postoperative patient.

Acute renal $ailure may develop in patients a$ter surgery $or several reasons. Althoughprerenal a2otemia is probably the most common cause o$ postoperative renal $ailure! urinarytract obstruction may occur a$ter pelvic surgery. Acute tubular necrosis due to to>ic! ischemic!or multi$actorial causes is also a common cause o$ acute renal $ailure in surgical patients.(his patient has rhabdomyolysis! which is a syndrome resulting $rom muscle injury. 3t can becaused by various traumatic and nontraumatic stressors in patients with and withoutunderlying muscle disease. (he disorder has been reported in patients who undergo surgery$or prolonged periods in the lithotomy position. (he $ull-blown syndrome is characteri2ed byhigh levels o$ circulating creatine inase! uric acid! potassium! and phosphorus! as a result o$the loss o$ muscle membrane integrity and the redistribution o$ these substances into thee>tracellular $luid. (he serum creatinine concentration increases and is o$ten disproportionatelyhigh compared with the increase in blood urea nitrogen. (he disproportionate increase results$rom decreased glomerular $iltration o$ creatinine and $rom muscle injury. 'ypocalcemia iso$ten present as a result o$ hyperphosphatemia and disorders in !#C-dihydro>yvitamin D and

parathyroid hormone metabolism and activity. An anion gap metabolic acidosis $re)uentlydevelops. (he hallmar o$ the syndrome is dipstic-positive heme due to myoglobinuria in theabsence o$ erythrocytes on con$irmatory microscopic e>amination.(his patient,s urine dipstic and microscopic e>amination $indings suggest that the diagnosis isnot due solely to urinary tract obstruction. (here is nothing in the urinalysis ?such as pyuria orleuocyte casts@ and no drugs are being administered to suggest a diagnosis o$ acuteinterstitial nephritis. (here is also nothing in the urinalysis ?such as protein! heme orerythrocyte casts@ to suggest a diagnosis o$ glomerulonephritis. (he high urinary sodiumconcentration and the abnormal microscopic e>amination argue against the diagnosis o$prerenal a2otemia.ibliography.<abrielli A! ;aruso +. "ostoperative acute renal $ailure secondary to rhabdomyolysis $rome>aggerated lithotomy position. 5 ;lin Anesth. 6669:#C7-8%.#./isweswaran "! <untupalli 5. =habdomyolysis. ;rit ;are ;lin. 6669C:C-#.

%.Oster 5=! 0inger 3! ;ontreras <N! Ahmad '3! /ieira ;. 4etabolic acidosis with e>tremeelevation o$ anion gap: case report and literature review. Am 5 4ed 0ci. 6669%7:%-6..0later 40! 4ullins =5. =habdomyolysis and myoglobinuric renal $ailure in trauma and surgicalpatients: a review. 5 Am ;oll 0urg. 6698:86%-78.C.ruce =<! 1im '! 4c=oberts F. =habdomyolysis and acute renal $ailure $ollowing radicalperineal prostatectomy. Urology. 66897:#7-%&.

Item 9

Page 5: Neurology Answers

8/11/2019 Neurology Answers

http://slidepdf.com/reader/full/neurology-answers 5/43

Answer: A"rotein o$ 8C&& mg*# hEducational Objective: =ecogni2e the #-hour urinary protein e>cretion associated withadvanced diabetic nephropathy.(his patient has many o$ the $eatures o$ advanced diabetic nephropathyMdiabetes mellitus $ormore than & years! diabetic retinopathy! hypertension! slowly progressive renal $ailure! andedema. (he typical patient with such $indings will have nephrotic-range proteinuria. "atients

with advanced diabetic nephropathy usually have urinary protein o$ greater than # to % g*# h.4icroalbuminuria ?urinary albumin o$ %& to %&& mg*# h@ occurs in patients with early diabeticnephropathy! but is not accompanied by a2otemia or nephrotic edema.ibliography.+arson (0. ;oncise review $or primary-care physicians: Evaluation o$ proteinuria. 4ayo ;lin"roc. 66986:C-.#.=it2 E! Orth 0=. Nephropathy in patients with type # diabetes mellitus. N Engl 5 4ed.6669%:#7-%%.%.reyer 5A! ain ="! Evans 51! Nahman N0 5r! +ewis E5! ;ooper 4! et al. "redictors o$ theprogression o$ renal insu$$iciency in patients with insulin-dependent diabetes and overtdiabetic nephropathy. (he ;ollaborative 0tudy <roup. 1idney 3nt. 6689C&:8C-.

Item 10

Answer: ;

+ow-salt diet and $urosemide with a goal o$ losing &.C-&.6 g ?-# lb@ dailyEducational Objective: Understand the treatment o$ edema in a patient with the nephroticsyndrome.(his patient re)uires both a low-salt diet and daily $urosemide with a goal o$ losing &.CJ&.6g ?J# lb@ daily. A low-salt diet alone is not ade)uate to reverse the avid sodium retention$ound in most patients with the nephrotic syndrome. +iewise! spironolactone alone will notcause enough natriuresis to achieve negative salt balance in severely nephrotic patients.(hese patients re)uire a loop diuretic! o$ten at a large dose! and at times the addition o$ asecond diuretic! such as metola2one! to develop a negative salt balance and lose edema $luid.0ince the edema is in the interstitial spaces and the diuretics all directly decrease the intra-vascular volume! a goal o$ losing only &.CJ&.6 g ?J# lb@ o$ edema $luid daily is desirable.4ore rapid natriuresis could lead to severe hypotension.ibliography.Orth 0=! =it2 E. Nephrotic syndrome. N Engl 5 4ed. 669%%:#&#-.

#.Appel <. <lomerulonephritis. 3n: <oldman +! ennett 5;! eds. ;ecil (e>tboo o$ 4edicine.#st ed. "hiladelphia: F 0aunders9 #&&&:C8-6. "ub4ed +in8

Item 11

Answer: ;(herapy should be started with both a low-saturated-$at! low-cholesterol diet and a %-hydro>y-%-methylglutaryl-coen2yme A ?'4<-;oA@ reductase inhibitorEducational Objective: =ecogni2e and treat the lipid abnormalities $ound in a patient with thenephrotic syndrome.(he dyslipidemia $ound in most patients with the nephrotic syndrome includes elevations o$plasma total cholesterol! low-density lipoprotein cholesterol! and lipoprotein?a@ levels. (he lipidabnormality has been indirectly lined to an increased ris o$ cardiovascular morbidity andmortality. oth dietary intervention and administration o$ a %-hydro>y-%-methylglutarylJcoen2yme A ?'4<J;oA@ reductase inhibitor are indicated. Dietary intervention may decrease

the levels o$ total and low-density lipoprotein cholesterol! but in patients with maredelevations o$ plasma lipids! diet alone is not ade)uate to normali2e these levels. Addition o$ areductase inhibitor ?statin@ drug can lead to mared reductions in lipid levels and is standardtherapy. Although lipoprotein?a@ levels are elevated in patients with the nephrotic syndromeand are a potential ris $actor $or cardiovascular disease! they do not respond to diet or to astatin drug. =emission o$ the proteinuria leads to a reduction o$ the lipoprotein?a@ levels.ibliography.Orth 0=! =it2 E. Nephrotic syndrome. N Engl 5 4ed. 669%%:#&#-.#.Ordone2 5D! 'iatt =A! 1illebrew E5! ireman '. (he increased ris o$ coronary heart diseaseassociated with the nephrotic syndrome. 1idney 3nt. 66%9:8%-#.

Page 6: Neurology Answers

8/11/2019 Neurology Answers

http://slidepdf.com/reader/full/neurology-answers 6/43

%.Fheeler D;! ernard D. +ipid abnormalities in the nephrotic syndrome9 causes!conse)uences! and treatment. Am 5 1idney Dis. 669#%:%%-8..Appel <. <lomerulonephritis. 3n: <oldman +! ennett 5;! eds. ;ecil (e>tboo o$ 4edicine.#st ed. "hiladelphia: F 0aunders9 #&&&:C8-6. "ub4ed +in8

Item 12

Answer: ;

p'Educational Objective: Understand that even subtle distal renal tubular acidosis can be thecause o$ recurrent calcium-containing idney stones.Distal renal tubular acidosis must be considered in all patients with recurring calcium-containing idney stones. (his patient,s biochemical studies showed a low serum bicarbonate!high serum chloride! and low serum potassium concentration! which are consistent with ahypoalemic! hyperchloremic metabolic acidosis. (he urine p' should be measured. 3$ the p'is elevated! this would help to establish a diagnosis o$ distal renal tubular acidosis. (he urinarycalcium:creatinine ratio may help clari$y the di$$erential diagnosis o$ hypercalciuria but wouldnot e>plain this patient,s abnormal serum chemistry values. (he patient has hypoalemia! anddetermining the transtubular potassium gradient could help e>plain why her potassiumconcentration is low! but would not address the low serum bicarbonate concentration orhistory o$ idney stones. Determination o$ $ractional e>cretion o$ sodium would not be help$ul.Urine culture would also not elucidate the cause o$ her recurrent idney stones ?not

withstanding the recent suggestion that many idney stones may be caused by [email protected].;oe + "ars 5'! Asplin 5=. (he pathogenesis and treatment o$ idney stones. N Engl 5 4ed.66#9%#7:-C#.#.renner =5! 0pring D! 0ebastian A! 4c0herry E4! <enant '1! "alubinsas A5! et al.3ncidence o$ radiographically evident bone disease! nephrocalcinosis! and nephrolithiasis invarious types o$ renal tubular acidosis. N Engl 5 4ed. 6#9%&7:#7-#.%.;i$tcioglu N! jorlund 4! 1uoriosi 1! ergstrom 1! 1ajander EO. Nanobacteria: anin$ectious cause $or idney stone $ormation. 1idney 3nt. 6669C8:6%-.

Item 13

Answer: ;Order thyroid-stimulating hormone determination

Educational Objective: =ecogni2e hypothyroidism as a secondary cause o$ hypertension.(his patient underwent a total thyroidectomy more than C years ago and has beennoncompliant about taing her thyroid e>tract replacement therapy! so that she has slowlybecome hypothyroid. 'er thyroid-stimulating hormone level was greater than C& mU*m+. (hecardiovascular e$$ects o$ hypothyroidism include a decrease in cardiac contractility! reduction inheart rate! and increased peripheral vascular resistance. 3ncreased peripheral vascularresistance may be due to an increase in circulating catecholamines secondary to increasedrelease and delayed metabolic clearance. Appro>imately #&G to &G o$ patients withhypothyroidism have hypertension despite having a decreased cardiac output. (hehypertension is predominantly diastolic with a diminished pulse pressure. "eriorbital edemaand nonpitting edema o$ the hands and $eet are characteristic. (he latter is due to interstitialaccumulation o$ glycosaminoglycans with associated water retention. Despite these $indings!intravascular volume contraction is o$ten present. Dyslipidemia is also common! with increasedtotal cholesterol and low-density-lipoprotein cholesterol concentrations.

Appropriate therapy $or this patient consists o$ cautious replacement o$ thyro>ine ?(@! whichshould ameliorate her hypertension and hyperlipidemia. Diuretic therapy may worsen herpresumed volume contraction! and b-blocer therapy may $urther reduce her heart rate. (heputative mild unilateral renal artery stenosis associated with a normal-si2ed idney is unlielyto be pathophysiologic when plasma renin activ-ity is suppressed. =enal angiography isthere$ore unnecessary.ibliography.1lein 3. (hyroid hormone e$$ects on the cardiovascular system. Am 5 4ed. 66&9:8%-7.#.0treeten D'! Anderson <' 5r! 'owland (! ;hiang =! 0mulyan '. E$$ects o$ thyroid $unctionon blood pressure. =ecognition o$ hypothyroid hypertension. 'ypertension. 69:7-%.

Page 7: Neurology Answers

8/11/2019 Neurology Answers

http://slidepdf.com/reader/full/neurology-answers 7/43

%.O,rien (! Dinneen 0! O,rien ";! "alumbo "5. 'yperlipidemia in patients with primary andsecondary hypo-thyroidism. 4ayo ;lin "roc. 66%98:8&-8..1lein 3! Ojamaa 1. (he cardiovascular system in hypothyroidism. 3n: raverman +E! Utiger+D! et al! eds. Ferner and 3ngbar,s (he (hyroid: A undamental and ;linical (e>t. 7th ed."hiladelphia: +ippincott-=aven9 668:766. U3: 6C7C.1lein 3! Ojamaa 1. (hyroid hormone and blood pressure regulation. 3n: +aragh 5'! renner4! eds. 'ypertension: "athophysiology! Diagnosis and 4anagement. #nd ed. New or:

=aven "ress +td9 66C:##7. U3: 6%C%7

Item 14

Answer: ADiscontinue indinavirEducational Objective: =ecogni2e the cause o$ acute renal $ailure in a young patient with '3/in$ection."atients with '3/ in$ection develop acute renal $ailure $or reasons similar to patients without'3/ in$ection. 0everal nephroto>ic medications are used in treating the superin$ections thatcomplicate the viral disease. 'ighly active antiretroviral therapy ?'AA=(@ may improve thecourse o$ patients with '3/-associated nephropathy. 'owever! recent reports have implicatedritonavir and indinavir as being associated with the development o$ acute renal $ailure inpatients with '3/ in$ection. 3ndinavir may cause acute renal $ailure as a result o$ intrinsic renaldisease due to unnown mechanisms. 3ndinavir may also be associated with postrenal

a2otemia because o$ its propensity to $orm crystals in the urine. 0everal urologic syndromeshave been associated with indinavir! including crystalluria! dysuria! and acute renal $ailure.(reatment with trimethoprimJsul$ametho>a2ole may increase the ris o$ developing renalinsu$$iciency. Abnormalities are usually reversible a$ter discontinuation o$ indinavir.Although captopril has been suggested as a treatment $or patients with '3/-associatednephropathy! this disorder is probably not present in this patient! who does not haveproteinuria. 3n addition! an angiotensin-converting en2yme inhibitor is contraindicated in apatient with acute renal $ailure. (here is no emergent indication $or dialysis in this patient.Although hyperalemia may develop! the patient,s serum potassium level may decrease i$ herrenal $unction improves a$ter cessation o$ the drug. 0tavudine has not been consistentlyassociated with the development o$ acute renal $ailure.ibliography.1opp 5! 4iller 1D! 4ican 5A! euerstein 34! /aughan E! aer ;! et al. ;rystalluria andurinary tract abnormalities associated with indinavir. Ann 3ntern 4ed. 6679#7:6-#C.

#.Fitte 4! (obon A! <ruenen$elder 5! <old$arb =! ;oburn 4. Anuria and acute renal $ailureresulting $rom indinavir sul$ate induced nephrolithiasis. 5 Urol. 669C6:6-6.%.1immel "+! osch 5"! /assalotti 5A. (reatment o$ human immunode$iciency virus ?'3/@-associated nephropathy. 0emin Nephrol. 669:8-C..erns 50! ;ohen =4! 0ilverman 4! (urner 5. Acute renal $ailure due to indinavir crystalluriaand nephrolithiasis: report o$ two cases. Am 5 1idney Dis. 6679%&:CC-8&.C."eraldi 4N! 4aslo ;! Aposso 1! 4ougenot ! =ondeau E! 0raer 5D. Acute renal $ailure in thecourse o$ '3/ in$ection: a single-institution retrospective study o$ ninety-two patients and si>tyrenal biopsies. Nephrol Dial (ransplant. 6669:C7-C.

Item 15

Answer: (he creatinine clearance is inaccurate because the #-hour urine collection is incompleteEducational Objective: 1now how to judge the accuracy o$ a #-hour urine collection.

(he best parameter $or judging the completeness o$ a #-hour urine collection is total urinarycreatinine e>cretion. 3$ the serum creatinine is stable! the #-hour urinary creatinine e>cretionmust e)ual the #-hour creatinine production ?$rom muscle@. 0tudies have shown that dailycreatinine production in men is 8 to #C mg*g and in women is C to #& mg*g ?the lowervalues $or each gender applying to the oldest aged individuals@. 3n this patient! who weighs g! his complete #-hour urine should contain #68 to #&#C mg o$ creatinine ?8 mg*g g P #68 mg*# hr and #C mg*g g P #&#C mg*# hr@. Even in a woman weighing g! the complete #-hour urine should contain between #C and 8#& mg o$ creatinine ?Cto #& mg*g@.

Page 8: Neurology Answers

8/11/2019 Neurology Answers

http://slidepdf.com/reader/full/neurology-answers 8/43

ibliography.Falser 4. ;reatinine e>cretion as a measure o$ protein nutrition in adults o$ varying age. 5"arenter Enteral Nutr. 679:7%0-0.

Item 16

Answer: D

;ryoglobulin determination and serologic tests $or hepatitis ;Educational Objective: =ecogni2e the importance o$ serum complement values in de$ining thepattern o$ glomerular disease.(his patient with progressive renal $ailure has erythrocyte casts! indicating a glomerular cause$or her disease. (he reduced total serum complement level and the greater reduction in thecomplement ; component compared with the complement ;% component are very help$ul inde$ining the cause o$ the glomerulonephritis. 3n patients with hepatitis ;Jrelatedmembranoproli$erative glomerulonephritis! the total complement levels are usually reduced!and the complement ; component is o$ten more greatly reduced than the ;% component.;ryoglobulins! cold precipitating antibodies! and immune comple>es are o$ten $ound in theserum o$ these patients and have been $ound to contain hepatitis ; =NA and anti-hepatitis ;antibodies ?anti-';/@. (he reduced serum complement level maes antineutrophil cytoplasmicantibodyJpositive ?AN;A-positive@ rapidly progressive glomerulonephritis and antiJglomerularbasement membrane ?anti-<4@ disease e>tremely unliely. 3t also e>cludes diseases

associated with abnormal immunoglobulins detected on electrophoresis! such as amyloidosisand light chain deposition disease. Although the total complement level is o$ten depressedduring the acute phase o$ immune comple> disease in patients with poststreptococcalglomerulonephritis! the complement ;% component is typically more depressed than thecomplement ; component. (here$ore! antistreptolysin O titer and antihyaluronidase assayare not needed.ibliography.Appel <. <lomerulonephritis. 3n: <oldman +! ennett 5;! eds. ;ecil (e>tboo o$ 4edicine.#st ed. "hiladelphia: F 0aunders9 #&&&:C8-6. "ub4ed +in8#.5ohnson =5! <retch D=! ;ouser F<! Alpers ;E! Filson 5! ;hung 4! et al. 'epatitis ; virus-associated glomerulonephritis. E$$ect o$ alpha-inter$eron therapy. 1idney 3nt. 6698:7&&-.%.Appel <! =adharishnan 5D! Agati /. 0econdary glomerular diseases. 3n: renner 4!=ector ;! eds. (he 1idney. 8th ed. "hiladelphia: F 0aunders9 #&&&. "ub4ed +in6

Item 17

Answer: D'er $etus is at ris $or intrauterine growth retardationEducational Objective: 1now how to counsel a pregnant woman with renal disease."regnant women with chronic renal insu$$iciency have a higher ris o$ giving birth to in$antswho sustained intrauterine growth retardation and to premature in$ants. "atients with chronicrenal disease who become pregnant also have a greater ris o$ developing preeclampsia thanwomen without renal insu$$iciency. Fomen with serum creatinine concentrations greater than. mg*d+ who become pregnant have a higher ris o$ developing increasing renalinsu$$iciency compared with women with elevated serum creatinine concentrations who do notbecome pregnant. Fomen with serum creatinine concentrations greater than #.& mg*d+ havea higher ?%%G@ ris o$ developing deteriorating renal $unction than women with less severerenal insu$$iciency ?#[email protected]

.'ou 0. "regnancy in chronic renal insu$$iciency and end-stage renal disease. Am 5 1idneyDis. 6669%%:#%C-C#.#.5ones D;! 'ayslett 5". Outcome o$ pregnancy in women with moderate or severe renalinsu$$iciency. N Engl 5 4ed. 6689%%C:##8-%#.

Item 18

Page 9: Neurology Answers

8/11/2019 Neurology Answers

http://slidepdf.com/reader/full/neurology-answers 9/43

Answer: Discontinue losartan and hydrochlorothia2ide*triamterene $or # wees9 obtain a #-hour urinespecimen $or sodium! potassium! and aldosterone e>cretion during oral salt-loadingEducational Objective: 3denti$y primary hyperaldosteronism as a secondary $orm o$hypertension in a patient with resistant hypertension and hypoalemia.(his patient,s abrupt onset o$ resistant hypertension and hypoalemia suggests a secondary$orm o$ hypertension. 4oreover! he is hypoalemic despite therapy with a potassium-sparing

diuretic and an angiotensin 33 receptor blocer and should be evaluated $or primary orsecondary hyper-aldosteronism. "atients with primary hyperaldosteronism present withhypertension and hypoalemia as a result o$ the sodium retentive and aliuretic properties o$unregulated aldosterone production $rom either adrenal adenomas or hyperplasia. (heoverproduction o$ aldosterone suppresses plasma renin activity. (he goal o$ the evaluation o$primary aldosteronism is two$old: @ to con$irm the biochemical abnormality o$ aldosteronehypersecretion! and #@ to locali2e the adrenal lesion. (he $ormer is best accomplished byobtaining a #-hour urine specimen $or aldosterone e>cretion! as the urinary aldosteronee>cretion rate appears to be the most sensitive and speci$ic biochemical test $or primaryhyperaldosteronism in the appropriately prepared patient. 3$ this test is positive! it should be$ollowed by thin-cut abdominal ;( to locali2e the lesion.0econdary hyperaldosteronism due to renal artery stenosis is less liely in this patient who hasuncontrolled hypertension and a normal renal scan and renal $unction while on an angiotensin33 receptor blocer. (here$ore! a captopril-stimulated renal scan is not indicated. An isolated

serum aldosterone determination is not indicated because this test is neither particularlysensitive nor speci$ic. A ratio o$ serum aldosterone:plasma renin activity is a better diagnostictest than a serum aldosterone determination alone! but whether this ratio is a betterdiagnostic study than a urinary aldosterone e>cretion rate in patients suspected o$ havingprimary hyperaldosteronism is controversial. 0imply administering potassium chloride withoutdiagnostic evaluation misses the opportunity to identi$y a correctable $orm o$ hypertension andhypoalemia.ibliography.ravo E+. 4anagement o$ hypocortisolism and hyperaldosteronism. 3n: 322o 5+ 5r! lac '=!<ood$iend (+! et al! eds. 'ypertension "rimer: (he Essentials o$ 'igh lood "ressure. #nd ed.altimore: Filliams Q Filins9666:6-C%. U3: 6#7C#.ravo E+. "heochromocytoma and mineralocorticoid hypertension. 3n: <lassoc =5! editor.;urrent (herapy in Nephrology and 'ypertension. th ed. 0t. +ouis: 4osby-ear oo! 3nc.669%%&-.

Item 19

Answer: ?@(he urinary uric acid:urinary creatinine ratio suggests that uric acid-mediated injury underliesthe pathologic processEducational Objective: Evaluate and treat acute renal $ailure in a patient with non-'odgin,slymphoma.0everal di$$erent types o$ renal disease may occur in patients with lymphoma. 3n this patientwith spontaneous tumor lysis syndrome! uric acid metabolic abnormalities ?con$irmed by thehigh urinary uric acid:urinary creatinine ratio@ mediate the renal $ailure. Urinary tractobstruction and prerenal a2otemia may occur be$ore or during treatment! and lymphomatousin$iltration o$ the idneys! associated with renal insu$$iciency! may rarely be an initialmani$estation o$ the disease.(umor lysis syndrome usually develops to % days a$ter initiating chemotherapy! especially in

patients in whom allopurinol is not given prophylactically. (he disease is more common inpatients with a large tumor burden! and the circulating levels o$ serum lactate dehydrogenaseand creatinine may be predictive o$ the complication.Although it is relatively uncommon! acute renal $ailure due to spontaneous tumor lysissyndrome is well recogni2ed. 3t occurs most o$ten in patients with poorly di$$erentiatedlymphoma! such as uritt,s lymphoma! and acute -cell lymphoblastic leuemia.0pontaneous tumor lysis syndrome has also been described in a patient with in$lammatorybreast cancer. (umor lysis syndrome causes acute renal $ailure associated with maredhyperuricemia. Acute uric acid nephropathy occurs because o$ precipitation o$ uric acid andresultant renal tubular obstruction. A urinary uric acid:urinary creatinine ratio o$ greater than

Page 10: Neurology Answers

8/11/2019 Neurology Answers

http://slidepdf.com/reader/full/neurology-answers 10/43

?normal less than &.8& to &.7C@ is suggestive o$ uric acid nephropathy.(he $ractional e>cretion o$ sodium o$ %.G in an oliguric patient is inconsistent with prerenala2otemia. Although we are not given a value $or the circulating creatine inase concentration!the absence o$ dipstic-positive heme on urinalysis! as well as the normal circulating serumphosphorus and potassium concentrations! maes rhabdomyolysis an unliely diagnosis. Adi$$erential $eature o$ spontaneous tumor lysis syndrome! compared with tumor lysissyndrome that develops a$ter therapy! is the absence o$ the typical hyperphosphatemia and

hypocalcemia that occur a$ter treatment ?perhaps because the neoplastic cells are able toincorporate [email protected] radiographic assessment to rule out urinary tract obstruction is usually indicated in patientswith acute renal $ailure and is o$ utmost importance in patients with an oncologic diagnosis.3maging studies can also be help$ul in suggesting a diagnosis o$ renal in$iltration by tumor.Administration o$ contrast agents! however! should only be undertaen $or an urgent indicationin patients with acute renal $ailure. (here are no data in this patient,s history or physicale>amination to suggest a diagnosis o$ renal arterial disease.(herapy a$ter the development o$ acute renal $ailure in patients with tumor lysis syndromeconsists o$ administration o$ allopurinol! care$ul administration o$ volume repletion! andprovision o$ dialysis as needed. Although alalini2ation o$ the urine may increase the solubilityo$ uric acid! it is not recommended $or patients with spontaneous or post-treatment tumorlysis syndrome because alalini2ing the urine is di$$icult in this setting! and an increasedurinary p' can predispose to precipitation o$ calcium phosphate.

4ost patients with tumor lysis syndrome have oliguria. (he renal disease has a goodprognosis! and the ultimate prognosis depends on the patient,s response to the oncologicdisease. Although there is no emergent indication $or renal replacement therapy in this patientat this time! dialysis is o$ten necessary to treat complications o$ acute renal $ailure such ashyperalemia that are not amenable to medical therapy.ibliography.0chelling 5=! <handour B! 0tricland (5! 0edor 5=. 4anagement o$ tumor lysis syndromewith standard continuous arteriovenous hemodialysis: case report and a review o$ theliterature. =en ail. 669#&:8%C-.#.Obrador <(! "rice ! O,4eara ! 0alant D5. Acute renal $ailure due to lymphomatousin$iltration o$ the idneys. 5 Am 0oc Nephrol. 6679:%-C.%.5ase A4! Day '5. Acute spontaneous tumor lysis syndrome. Am 5 'ematol. 6697:#6-%..'ande 1=! <arrow <;. Acute tumor lysis syndrome in patients with high-grade non-

'odgin,s lymphoma. Am 5 4ed. 66%96:%%-6.C.'ain =D! 'arvey E! "oon AO! Feit2man 0. Acute tumour lysis syndrome with no evidence o$tumour load. "ediatr Nephrol. 669:C%7-6.

Item 20

Answer: ?D@=enal arteriographyEducational Objective: Evaluate and treat acute renal $ailure in a patient with non-'odgin,slymphoma.0everal di$$erent types o$ renal disease may occur in patients with lymphoma. 3n this patientwith spontaneous tumor lysis syndrome! uric acid metabolic abnormalities ?con$irmed by thehigh urinary uric acid:urinary creatinine ratio@ mediate the renal $ailure. Urinary tractobstruction and prerenal a2otemia may occur be$ore or during treatment! and lymphomatousin$iltration o$ the idneys! associated with renal insu$$iciency! may rarely be an initial

mani$estation o$ the disease.(umor lysis syndrome usually develops to % days a$ter initiating chemotherapy! especially inpatients in whom allopurinol is not given prophylactically. (he disease is more common inpatients with a large tumor burden! and the circulating levels o$ serum lactate dehydrogenaseand creatinine may be predictive o$ the complication.Although it is relatively uncommon! acute renal $ailure due to spontaneous tumor lysissyndrome is well recogni2ed. 3t occurs most o$ten in patients with poorly di$$erentiatedlymphoma! such as uritt,s lymphoma! and acute -cell lymphoblastic leuemia.0pontaneous tumor lysis syndrome has also been described in a patient with in$lammatorybreast cancer. (umor lysis syndrome causes acute renal $ailure associated with mared

Page 11: Neurology Answers

8/11/2019 Neurology Answers

http://slidepdf.com/reader/full/neurology-answers 11/43

hyperuricemia. Acute uric acid nephropathy occurs because o$ precipitation o$ uric acid andresultant renal tubular obstruction. A urinary uric acid:urinary creatinine ratio o$ greater than ?normal less than &.8& to &.7C@ is suggestive o$ uric acid nephropathy.(he $ractional e>cretion o$ sodium o$ %.G in an oliguric patient is inconsistent with prerenala2otemia. Although we are not given a value $or the circulating creatine inase concentration!the absence o$ dipstic-positive heme on urinalysis! as well as the normal circulating serumphosphorus and potassium concentrations! maes rhabdomyolysis an unliely diagnosis. A

di$$erential $eature o$ spontaneous tumor lysis syndrome! compared with tumor lysissyndrome that develops a$ter therapy! is the absence o$ the typical hyperphosphatemia andhypocalcemia that occur a$ter treatment ?perhaps because the neoplastic cells are able toincorporate [email protected] radiographic assessment to rule out urinary tract obstruction is usually indicated in patientswith acute renal $ailure and is o$ utmost importance in patients with an oncologic diagnosis.3maging studies can also be help$ul in suggesting a diagnosis o$ renal in$iltration by tumor.Administration o$ contrast agents! however! should only be undertaen $or an urgent indicationin patients with acute renal $ailure. (here are no data in this patient,s history or physicale>amination to suggest a diagnosis o$ renal arterial disease.(herapy a$ter the development o$ acute renal $ailure in patients with tumor lysis syndromeconsists o$ administration o$ allopurinol! care$ul administration o$ volume repletion! andprovision o$ dialysis as needed. Although alalini2ation o$ the urine may increase the solubilityo$ uric acid! it is not recommended $or patients with spontaneous or post-treatment tumor

lysis syndrome because alalini2ing the urine is di$$icult in this setting! and an increasedurinary p' can predispose to precipitation o$ calcium phosphate.4ost patients with tumor lysis syndrome have oliguria. (he renal disease has a goodprognosis! and the ultimate prognosis depends on the patient,s response to the oncologicdisease. Although there is no emergent indication $or renal replacement therapy in this patientat this time! dialysis is o$ten necessary to treat complications o$ acute renal $ailure such ashyperalemia that are not amenable to medical therapy.ibliography.0chelling 5=! <handour B! 0tricland (5! 0edor 5=. 4anagement o$ tumor lysis syndromewith standard continuous arteriovenous hemodialysis: case report and a review o$ theliterature. =en ail. 669#&:8%C-.#.Obrador <(! "rice ! O,4eara ! 0alant D5. Acute renal $ailure due to lymphomatousin$iltration o$ the idneys. 5 Am 0oc Nephrol. 6679:%-C.%.5ase A4! Day '5. Acute spontaneous tumor lysis syndrome. Am 5 'ematol. 6697:#6-

%..'ande 1=! <arrow <;. Acute tumor lysis syndrome in patients with high-grade non-'odgin,s lymphoma. Am 5 4ed. 66%96:%%-6.C.'ain =D! 'arvey E! "oon AO! Feit2man 0. Acute tumour lysis syndrome with no evidence o$tumour load. "ediatr Nephrol. 669:C%7-6. K

Item 21

Answer: 0erum uric acid o$ &.C mg*d+Educational Objective: =ecogni2e the ris $actors $or calcium nephrolithiasis.'ypercalciuria! hypocitraturia! and hypero>aluria are all ris $actors $or calcium stone disease.(he presence o$ hypercalcemia and hypophosphatemia suggests the diagnosis o$hyperparathyroidismMa condition in which %&G o$ patients will have calcium nephrolithiasis.Although increased urinary uric acid e>cretion is a ris $actor $or calcium nephrolithiasis! an

elevated serum uric acid level alone is not.ibliography.ushinsy DA. Nephrolithiasis. 5 Am 0oc Nephrol. 669 6:67-#.

Item 22

Page 12: Neurology Answers

8/11/2019 Neurology Answers

http://slidepdf.com/reader/full/neurology-answers 12/43

Answer: ;ocal glomerulosclerosis is the most liely diagnosis because it is the most common pattern o$idiopathic nephrotic syndrome in blac Americans! and treatment with a prolonged course o$glucocorticoids is success$ul in up to C&G o$ patientsEducational Objective: Understand the $re)uency and potential treatment o$ the di$$erentpatterns o$ idiopathic nephrotic syndrome in adults.ocal glomerulosclerosis is the most common pattern o$ idiopathic nephrotic syndrome in blac

American patients. Although idiopathic $ocal segmental glomerulosclerosis was $ormerlythought to be an untreatable lesion! it now appears that up to C&G o$ patients will respond toglucocorticoids or other drugs such as cytoto>ic agents or cyclosporine. 4embranousnephropathy is still the most common cause o$ idiopathic nephrotic syndrome in whiteAmerican patients and hence in all American patients when racial $actors are not considered. Asigni$icant percentage o$ patients respond to some $orm o$ immunosuppressive therapy! butthe e>act number is debated. Although patients with minimal change disease typically havenormal renal $unction! so do a majority o$ patients with other $orms o$ idiopathic nephroticsyndrome. (he vast majority o$ patients with minimal change disease will have completeremission o$ proteinuria with glucocorticoid therapy. As many as &G o$ patients over age 8&with the nephrotic syndrome will be $ound to have A+ amyloidosis9 however! it is still anuncommon cause o$ the nephrotic syndrome! especially in patients less than 8& years o$ age.;hemotherapy with melphalan or cyclophosphamide! along with glucocorticoids! o$ten leads toimproved renal $unction.

ibliography.'aas 4! 4eehan 04! 1arrison (<! 0pargo '. ;hanging etiologies o$ une>plained adultnephrotic syndrome: a comparison o$ renal biopsy $indings $rom 678-676 and 66C-667.Am 5 1idney Dis. 6679%&:8#-%.#.Appel <. <lomerulonephritis. 3n: <oldman +! ennett 5;! eds. ;ecil (e>tboo o$ 4edicine.#st ed. "hiladelphia: F 0aunders9 #&&&:C8-6.%.4atalon A! /aleri A! Appel <. (reatment o$ $ocal segmental glomerulosclerosis. 0eminNephrol. #&&&9#&:%&6-7..<eddes ;;! ;attran D;. (he treatment o$ idiopathic membranous nephropathy. 0eminNephrol. #&&&9#&:#66-%&.

Item 23

Answer: 0urreptitious vomiting

Educational Objective: Understand the diagnostic importance o$ a low urinary chlorideconcentration in the di$$erential diagnosis o$ metabolic alalosis.(his patient has a serum bicarbonate o$ % me)*+ and a serum potassium o$ #. me)*+! whichare almost certainly due to metabolic alalosis. 0urreptitious vomiting ?which was latercon$irmed in this patient@ must be considered in any patient with otherwise une>plainedmetabolic alalosis. A low urinary chloride concentration points strongly toward this diagnosis.(he urinary electrolyte analysis is very help$ul in this setting because the low urinary chlorideconcentration suggests vomiting! whereas a high urinary chloride concentration is moreconsistent with a mineralocorticoid or a mineralocorticoid-lie e>cess syndrome! artter,ssyndrome! or <itelman,s syndrome. =ecent studies have shown that classic artter,s syndromeis o$ten due to a genetic mutation in the Na-1-#;l transporter located in the thic ascendinglimb o$ 'enle. (his is the transporter that is inhibited by loop diuretics. <itelman,s syndromehas a similar clinical presentation but is due to a genetic de$ect in the neutral Na-;ltransporter in the distal tubule. (hia2ide diuretics inhibit this transporter. (hese two disorders

are clinically distinguished by the presence o$ hypercalciuria in artter,s syndrome and hypo-calciuria in <itelman,s syndrome. "atients with <itelman,s syn-drome also usually have moresevere hypomagnesemia. A high urinary chloride concentration is also consistent with recentuse o$ diuretics. Distinguishing between artter,s and <itelman,s syndromes and surreptitiousdiuretic ingestion may be very di$$icult. 'ypoalemic periodic paralysis does not present in thismanner and is not typically associated with metabolic alalosis or a low urinary chlorideconcentration.

ibliography

Page 13: Neurology Answers

8/11/2019 Neurology Answers

http://slidepdf.com/reader/full/neurology-answers 13/43

."almer ! Alpern =5. 4etabolic alalosis. 5 Am 0oc Nephrol. 6679:8#-6.#."ere2 <O! Oster 5=! =ogers A. Acid-base disturbances in gastrointestinal disease. Dig Dis0ci. 679%#:&%%-%.%.4itchell 5E! 0eim ';! ;olon E! "omeroy ;. 4edical complications and medical managemento$ bulimia. Ann 3ntern 4ed. 679&7:7-7..<uay-Food$ord +4. artter syndrome: unraveling the pathophysiologic enigma. Am 5 4ed.669&C:C-8.

Item 24

Answer: ;hange to another antihypertensive agent and increase the dose until her blood pressure is K&*6& mm 'gEducational Objective: =ecogni2e the importance o$ achieving target blood pressure with aminimum number o$ side e$$ects.(his patient,s blood pressure control while on amlodipine is inade)uate ?blood pressure hasnot been reduced to K &*6& mm 'g@! and she has developed pedal edema attributable tothe dihydropyridine calcium channel blocer. 'er medication should be changed to anotherantihypertensive agent that is unliely to induce edema and will optimi2e blood pressurecontrol. As noted in the report o$ the 0i>th 5oint National ;on$erence on "revention! Detection!Evaluation! and (reatment o$ 'igh lood "ressure ?5N;-/3@! ade)uate control o$ blood pressureto K &*6& mm 'g is achieved in only CG o$ patients treated with medication! which

represents only #7G o$ all patients with hypertension. (his has been labeled the Rgreathypertension disconnect.S Almost all physicians now the target blood pressure o$ K &*6&mm 'g9 however! we are not very success$ul in achieving this target. Fhen target bloodpressure is not attained! the 5N;-/3 recommends three possible options: @ increase the doseo$ the initial agent! #@ add a second agent! or %@ change to another drug or class o$ agent.(his patient re)uires a change to another antihypertensive agent to achieve a target bloodpressure and reduce side e$$ects. (wo recent studies have documented that physicians o$ten$ail to modi$y therapy when patients have not achieved a target blood pressure. 3ncreasing thispatient,s dihydro-pyridine calcium channel blocer is not indicated because this will lielyincrease her edema. inally! salt restriction and support hose are unliely to resolve themedication-induced edema.ibliography.(he si>th report o$ the 5oint National ;ommittee on "revention! Detection! Evaluation and(reatment o$ 'igh lood "ressure. Arch 3ntern 4ed. 6679C7:#%-8.

#.erlowit2 D=! Ash A0! 'icey E;! riedman ='! <licman 4! 1ader ! et al. 3nade)uatemanagement o$ blood pressure in a hypertensive population. N Engl 5 4ed. 669%%6:6C7-8%.

Item 25

Answer: ?;@Administer intravenous hal$-normal saline be$ore per$orming angiographyEducational Objective: "revent and treat contrast-induced renal $ailure in an elderlyoutpatient with chronic renal disease."atients with renal insu$$iciency have a higher incidence o$ renal $ailure a$ter e>posure tocontrast agents! a syndrome nown as contrast nephropathy. (he ris is $urther increased inpatients with concurrent diabetes mellitus and renal insu$$iciency. Use o$ nonionic and low-osmolality contrast media may be bene$icial in high-ris patients with renal insu$$iciency.Administration o$ intravenous $luids ?most o$ten hal$-normal saline@ be$ore the radiographic

procedure has been associated with a decreased incidence o$ contrast nephropathy in patientswith di$$erent ris $actors. 3ntravenous $luids appear to be more e$$ective than $urosemide andmannitol! which are there$ore not currently recommended. Administration o$ dopamine hasbeen associated with poorer outcomes in patients with diabetes mellitus who have beene>posed to contrast media. 0ome studies have suggested a role $or calcium channel blocersin reducing the incidence o$ contrast nephropathy in selected patients. (here$ore!discontinuing diltia2em in this patient is not recommended. Use o$ acetylcysteine be$oreadministration o$ contrast has been shown to limit the occurrence o$ deterioration o$ renal$unction in a recent study! but the clinical characteristics o$ the population that should receivesuch an intervention and the ris*bene$it analysis have not yet been delineated.

Page 14: Neurology Answers

8/11/2019 Neurology Answers

http://slidepdf.com/reader/full/neurology-answers 14/43

No therapy has been shown to be e$$ective in ameliorating the course o$ established contrastnephropathy. 3n the absence o$ a well-designed clinical trial to address this issue! the standardo$ care remains observation! avoidance o$ nephroto>ins! and provision o$ renal replacementtherapy $or speci$ic indications! such as hyperalemia! acidemia! and congestive heart $ailurenot manageable by medical means. Dopamine has been shown to worsen the course o$patients with established contrast nephropathy a$ter coronary angiography. =igorouslycontrolled clinical trials have shown that dialysis to remove contrast does not improve the

course o$ patients with contrast nephropathy.ibliography.ehrend (! 4iller 0. Acute renal $ailure in the cardiac care unit: etiologies! outcomes! andprognostic $actors. 1idney 3nt. 6669C8:#%-%.#.4urphy 0F! arrett 5! "ar$rey "0. ;ontrast nephropathy. 5 Am 0oc Nephrol. #&&&9:77-#.%.0olomon =. =adiocontrast-induced nephropathy. 0emin Nephrol. 669:CC-7..=udnic 4=! <old$arb 0! Fe>ler +! +udbroo "A! 4urphy 45! 'alpern E! et al. Nephroto>icityo$ ionic and nonionic contrast media in 68 patients: a randomi2ed trial. (he 3ohe>ol;ooperative 0tudy. 1idney 3nt. 66C97:#C-8.C.(epel 4! van der <eit 4! 0chwar2$eld ;! +au$er U! +iermann D! Bide F. "revention o$radiographic-contrast-agent-induced reductions in renal $unction by acetylcysteine. N Engl 54ed. #&&&9%%:#&-#.

Item 26

Answer: ?A@4onitor9 begin renal replacement therapy i$ indicatedEducational Objective: "revent and treat contrast-induced renal $ailure in an elderlyoutpatient with chronic renal disease."atients with renal insu$$iciency have a higher incidence o$ renal $ailure a$ter e>posure tocontrast agents! a syndrome nown as contrast nephropathy. (he ris is $urther increased inpatients with concurrent diabetes mellitus and renal insu$$iciency. Use o$ nonionic and low-osmolality contrast media may be bene$icial in high-ris patients with renal insu$$iciency.Administration o$ intravenous $luids ?most o$ten hal$-normal saline@ be$ore the radiographicprocedure has been associated with a decreased incidence o$ contrast nephropathy in patientswith di$$erent ris $actors. 3ntravenous $luids appear to be more e$$ective than $urosemide andmannitol! which are there$ore not currently recommended. Administration o$ dopamine hasbeen associated with poorer outcomes in patients with diabetes mellitus who have been

e>posed to contrast media. 0ome studies have suggested a role $or calcium channel blocersin reducing the incidence o$ contrast nephropathy in selected patients. (here$ore!discontinuing diltia2em in this patient is not recommended. Use o$ acetylcysteine be$oreadministration o$ contrast has been shown to limit the occurrence o$ deterioration o$ renal$unction in a recent study! but the clinical characteristics o$ the population that should receivesuch an intervention and the ris*bene$it analysis have not yet been delineated.No therapy has been shown to be e$$ective in ameliorating the course o$ established contrastnephropathy. 3n the absence o$ a well-designed clinical trial to address this issue! the standardo$ care remains observation! avoidance o$ nephroto>ins! and provision o$ renal replacementtherapy $or speci$ic indications! such as hyperalemia! acidemia! and congestive heart $ailurenot manageable by medical means. Dopamine has been shown to worsen the course o$patients with established contrast nephropathy a$ter coronary angiography. =igorouslycontrolled clinical trials have shown that dialysis to remove contrast does not improve thecourse o$ patients with contrast nephropathy.

ibliography.ehrend (! 4iller 0. Acute renal $ailure in the cardiac care unit: etiologies! outcomes! andprognostic $actors. 1idney 3nt. 6669C8:#%-%.#.4urphy 0F! arrett 5! "ar$rey "0. ;ontrast nephropathy. 5 Am 0oc Nephrol. #&&&9:77-#.%.0olomon =. =adiocontrast-induced nephropathy. 0emin Nephrol. 669:CC-7..=udnic 4=! <old$arb 0! Fe>ler +! +udbroo "A! 4urphy 45! 'alpern E! et al. Nephroto>icityo$ ionic and nonionic contrast media in 68 patients: a randomi2ed trial. (he 3ohe>ol;ooperative 0tudy. 1idney 3nt. 66C97:#C-8.C.(epel 4! van der <eit 4! 0chwar2$eld ;! +au$er U! +iermann D! Bide F. "revention o$

Page 15: Neurology Answers

8/11/2019 Neurology Answers

http://slidepdf.com/reader/full/neurology-answers 15/43

radiographic-contrast-agent-induced reductions in renal $unction by acetylcysteine. N Engl 54ed. #&&&9%%:#&-#.

Item 27

Answer: ;Uric acid stone in the right renal pelvisEducational Objective: =ecogni2e the radiographic and clinical $eatures o$ uric acid stone

disease.'ematuria and $lan pain can be associated with any o$ the mentioned disorders. (he spiral ;(without contrast shows the typical $eatures o$ a stone in the right renal pelvis. othradiopa)ue ?calcium-containing@ and radiolucent ?uric acid@ stones will be visuali2ed on a ;(scan without contrast. A uric acid stone is typically radiolucent ?invisible@ on plain radiographsand is easily seen on ;( scans. 'owever! a .C-cm calcium stone in the renal pelvis should bevisible on the plain radiograph. (he very acid urinary p' is also typical o$ uric acid stonedisease. (ransitional cell carcinoma! analgesic nephropathy! and chronic pyelonephritis are notusually associated with nephrolithiasis.ibliography.+ow =1! 0toller 4+. Uric acid-related nephrolithiasis. Urol ;lin North Am. 6679#:%C-.

Item 28

Answer: ;

AmyloidosisEducational Objective: =ecogni2e the clinical presentation o$ amyloid renal disease.(his patient has nephrotic-range proteinuria! renal dys$unction! and a signi$icantly abnormalmonoclonal protein in his serum. Although his history and laboratory data ?including the lowvoltage on the electrocardiogram@ are compatible with both amyloidosis and light chaindeposition disease! amyloidosis is by $ar the more common disorder. 4oreover! &G o$patients with amyloidosis have a lambda light chain protein abnormality! whereas &G o$patients with light chain deposition disease have a appa light chain protein abnormality.Neither o$ these $indings would be e>pected in patients with membranous nephropathy.4yeloma cast nephropathy causes renal dys$unction and is associated with signi$icantimmunoglobulin light chain proteinuria and tubular disease. 3t is not associated with heavyalbuminuria.ibliography.Appel <. <lomerulonephritis. 3n: <oldman +! ennett 5;! eds. ;ecil (e>tboo o$ 4edicine.

#st ed. "hiladelphia: F 0aunders9 #&&&:C8-6.#.Appel <! =adharishnan 5D! Agati /. 0econdary glomerular diseases. 3n: renner 4!=ector ;! eds. (he 1idney. 8th ed. "hiladelphia: F 0aunders9 #&&&.%.Finearls ;<. Acute myeloma idney. 1idney 3nt. 66C9 :%7-8.

Item 29

Answer: ;%C to C me)*+Educational Objective: Understand how hyperglycemia causes the serum sodiumconcentration to $all and how to calculate the e>pected serum sodium concentration a$ter thehyperglycemia has been corrected.(he serum sodium concentration is most liely to be %C to C me)*+. 'yperglycemia causesa shi$t o$ water $rom the intracellular space into the e>tracellular space. (his simultaneouslydilutes the e>tracellular sodium salts and increases the intracellular solute concentration. (he

water shi$t continues until the e>tracellular and intracellular osmolalities are similar. 3t cangenerally be assumed that in adults each && mg*d+ increase in plasma glucose will cause awater shi$t that reduces the serum sodium concentration by appro>imately .8 me)*+.;onse)uently! i$ the plasma glucose increases $rom && to #&& mg*d+! this should reduce theserum sodium concentration by about me)*+. ;onversely! i$ insulin reduces the plasmaglucose $rom #&& to && mg*d+! the serum sodium concentration should increase by about me)*+ $rom # to #. (here$ore! this patient,s hyponatremia can be entirely e>plained bya shi$t o$ water $rom the intracellular space to the e>tracellular space! and a$ter correction o$hyperglycemia! her serum sodium concentration should be restored to the normal range.Although one recent study suggests the correction $actor may be higher than .8! perhaps as

Page 16: Neurology Answers

8/11/2019 Neurology Answers

http://slidepdf.com/reader/full/neurology-answers 16/43

high as #. me)*+! this is a single study o$ acute hyperglycemia in otherwise normal subjectsand has not yet been con$irmed. (he applicability o$ the new data to diabetic patients withhyperglycemia remains unclear.ibliography.1at2 4A. 'yperglycemia-induced hyponatremiaJcalculation o$ e>pected serum sodiumdepression. N Engl 5 4ed. 67%9#6:%-.#.Daugirdas 5(! 1ron$ol NO! (2amalouas A'! 3ng (0. 'yperosmolar coma: cellular

dehydration and the serum sodium concentration. Ann 3ntern 4ed. 669&:CC-7.%."opli 0! +eehey D5! Daugirdas 5(! ansal /1! 'o D0! 'ano 5E! et al. Asymptomatic!nonetotic severe hyperglycemia with hyponatremia. Arch 3ntern 4ed. 66&9C&:68#-..'illier (A! Abbott =D! arrett E5. 'yponatremia: evaluating the correction $actor $orhyperglycemia. Am 5 4ed. 6669 &8:%66-&%.

Item 30

Answer: ;=enal transplantationEducational Objective: =ecogni2e the advantages o$ renal transplantation in managingpatients with diabetic renal disease.=ecent data comparing survival o$ patients with renal disease who were matched $or age andtype o$ disease have shown a clear survival advantage $or renal transplantation compared withdialysis. (his is particularly true $or patients with diabetes mellitus! as transplantation o$$ers

both a survival advantage and improved )uality o$ li$e. "rior studies comparing patients withdiabetes mellitus and renal disease who were treated with transplantation versus dialysis havebeen $lawed by selection bias! as not all patients with diabetes are medically )uali$ied $ortransplantation. A recent study o$ patients with diabetes mellitus and end-stage renal diseasedemonstrated increased survival $or patients who received a renal transplant compared withthose who had been cleared $or transplantation but remained on the waiting list. <iven thissurvival advantage! patients with progressive chronic renal $ailure should be re$erred early $orconsideration $or either cadaveric or living-related donor renal transplantation. 3$ possible! aliving-related donor transplant is most liely the best available option $or this patient.;ombined renal and pancreas transplantation is not indicated because patients with type #diabetes mellitus are not considered candidates $or a pancreas transplant. Dialysis is notpre$erred long-term renal replacement therapy $or this patient. 3$ dialysis must be used! thechoice o$ hemodialysis or perito-neal dialysis is usually based upon availability! li$estyle! need$or independence! and access! as there is no clearcut survival advantage between dialysis

modalities $or this type o$ patient.ibliography."ort 1! Fol$e =A! 4auger EA! erling D"! 5iang 1. ;omparison o$ survival probabilities $ordialysis patients vs cadaveric renal transplant recipients. 5A4A. 66%9#7&:%%6-%.#.loembergen FE! "ort 1! 4auger EA! Fol$e =A. A comparison o$ mortality between patientstreated with hemodialysis and peritoneal dialysis. 5 Am 0oc Nephrol. 66C98:77-%.%.Fol$e =A! Ashby /! 4il$ord E+! Ojo AO! Ettenger =E! Agodoa +! et al. ;omparison o$mortality in all patients on dialysis! patients on dialysis awaiting transplantation! and recipientso$ a $irst cadaveric transplant. N Engl 5 4ed. 6669%:7#C-%&..'ariharan 0! 5ohnson ;"! resnahan A! (aranto 0E! 4c3ntosh 45! 0tablein D. 3mprovedgra$t survival a$ter renal transplantation in the United 0tates! 6 to 668. N Engl 5 4ed.#&&&9%#:8&C-#.C.Bimmerman 0F! 0ollinger '! Faeen 4! Armbrust 4! ;ole D! 1earney 4E! et al. =enalreplacement therapy in diabetic nephropathy. Adv =en =eplace (her. 669:88-7.

Item 31

Page 17: Neurology Answers

8/11/2019 Neurology Answers

http://slidepdf.com/reader/full/neurology-answers 17/43

Answer: ;Nephropathy due to ;hinese herbsEducational Objective: Evaluate acute renal $ailure in a patient taing an alternativemedicine.(he change in serum creatinine concentration $orm &.7 mg*d+ to .6 mg*d+ over a relativelyshort period o$ time establishes the diagnosis o$ acute renal $ailure. ;hinese herbs andmedicines have been associated with several renal syndromes. (hese medications can be

prescribed $or various reasons! including protection $rom idney disease! treatment o$hypertension and dermatologic diseases! and weight reduction. ;hinese medicines obtainedabroad may be adulterated with anti-in$lammatory drugs or nephroto>ins such as cadmium!which can be associated with several types o$ renal $ailure."atients with nephropathy due to ;hinese herbs usually have renal insu$$iciency with normalblood pressure! anemia! and mild tubular proteinuria. (he urinalysis is o$ten normal. 0everalpatients evaluated in Europe were treated with ;hinese herb preparations containing0tephania tetranda and 4agnolia o$$icinalis to induce weight loss. Other data implicatearistolochic acid in herbal preparations as a nephroto>in. 3n most patients! renal $ailureprogresses to end-stage renal disease over months to years. iopsy specimens show renaltubular disease and interstitial $ibrosis without a prominent immune cell in$iltrate. 4arers o$pro>imal tubular injury may play an important prognostic role in the assessment o$ suchpatients. Although some patients have been treated with glucocorticoids with a possiblebene$icial e$$ect! the e$$icacy o$ these drugs has not been e>amined in rigorously controlled

clinical trials.Diabetic nephropathy can complicate type # diabetes mellitus9 however! there is usually a longperiod o$ poor glucose control associated with the development o$ diabetic nephropathy in asusceptible patient. 3n addition! although patients with diabetic renal disease may sustainacute renal $ailure $rom other causes! renal insu$$iciency $rom diabetic nephropathy does notprogress at such a rapid pace. (he abnormal urinalysis $indings argue against a diagnosis o$prerenal a2otemia! and the lac o$ signi$icant proteinuria! hematuria! or erythrocyte castsmaes the diagnosis o$ 3gA nephropathy unliely.ibliography.<oldman 5A! 4yerson <. ;hinese herbal medicine: camou$laged prescriptionantiin$lammatory drugs! corticosteroids! and lead. Arthritis =heum. 669%:#&7.#./anherweghem 5+! Depierreu> 4! (ielemans 0! Abramowic2 D! Dratwa 4! 5adoul 4! et al.=apidly progressive interstitial renal $ibrosis in young women: association with slimmingregimen including ;hinese herbs. +ancet. 66%9%:%7-6.

%."illans "3. (o>icity o$ herbal products. N B 4ed 5. 66C9&:86-7..van persele de 0trihou ;! /anherweghem 5+. (he tragic paradigm o$ ;hinese herbsnephropathy. Nephrol Dial (ransplant. 66C9C7-8&.C.Nortier 5+! Deschodt-+ancman 44! 0imon 0! (hielemans NO! de "re2 E<! Depierreu> 4! etal. "ro>imal tubular injury in ;hinese herbs nephropathy: monitoring by neutral endopeptidaseen2ymuria. 1idney 3nt. 6679C:#-6%.

Item 32

Answer: A=enal ultrasonographyEducational Objective: =ecogni2e the clinical and laboratory $eatures o$ '3/-associatednephropathy.(his patient! who is in$ected with '3/! has massive proteinuria and renal insu$$iciency. '3/-associated nephropathy is there$ore a liely diagnosis. Although a conclusive diagnosis can

only be made by renal biopsy! most patients with '3/-nephropathy are blac Americans whopresent with heavy proteinuria! the nephrotic syndrome! and renal dys$unction.Ultrasonography is the most appropriate initial study. 3t typically shows the large echogenicidneys associated with '3/ nephropathy and is highly supportive o$ the diagnosis. (hecontrast agents used $or intravenous pyelography and renal angiography are relativelycontraindicated! and these procedures would not help establish the diagnosis.Echocardiography is not use$ul in de$ining the cause o$ most glomerular diseases.

ibliography

Page 18: Neurology Answers

8/11/2019 Neurology Answers

http://slidepdf.com/reader/full/neurology-answers 18/43

.D,Agati /D! Appel <. '3/ in$ection and the idney. 5 Am 0oc Nephrol. 6679:%-C#.#.D,Agati /D! Appel <. =enal pathology o$ human immunode$iciency virus in$ection. 0eminNephrol. 669:&8-#.%.'umphreys! 4'. 'uman immunode$iciency virus-associated glomerulosclerosis. 1idney 3nt.66C9:%-#&..Finston 5A! urns <;! 1lotman "E. (he human immunode$iciency virus ?'3/@ epidemic and'3/-associated nephropathy. 0emin Nephrol. 669:%7%-77.

C.1lotman "E. '3/ associated nephropathy. 1idney 3nt. 6669C8:8-78.

Item 33

Answer: AAn angiotensin-converting en2yme inhibitorEducational Objective: 1now the most appropriate renal therapy $or a patient with '3/-associated nephropathy.(his patient,s renal biopsy shows classic $eatures o$ '3/-associated nephropathy. (hecollapsing pattern o$ $ocal glomerulosclerosis is an uncommon pattern in patients who haveidiopathic $ocal glomerulosclerosis but is typically present in patients with '3/-associatednephropathy and e>treme dilatation o$ the renal tubules. Electron microscopy o$ten showstubuloreticular structures within the endothelial cells. =ecent studies have con$irmed that useo$ angiotensin-converting en2yme inhibitors in patients with classic '3/-associatednephropathy not only decreases the proteinuria but also slows the progression o$ the disease.

(here is no proven role $or an immunosuppressive agent such as pulse methylprednisolone orcyclophosphamide in patients with this disorder. (hese agents may lead to an increased ris o$ opportunistic in$ections in patients with '3/ in$ection. ;alcium channel blocers have not beenshown to reduce proteinuria or progression to renal $ailure in patients with '3/-associatednephropathy.ibliography.D,Agati /D! Appel <. '3/ in$ection and the idney. 5 Am 0oc Nephrol. 6679:%-C#.#.D,Agati /D! Appel <. =enal pathology o$ human immunode$iciency virus in$ection. 0eminNephrol. 669:&8-#.%.1lotman "E. '3/ associated nephropathy. 1idney 3nt. 6669C8:8-78..1immel "+! 4ishin <5! Umana FO. ;aptopril and renal survival in patients with humanimmunode$iciency virus nephropathy. Am 5 1idney Dis. 6689#:#&#-.C.Finston 5A! urns <;! 1lotman "E. (reatment o$ '3/-associated nephropathy. 0eminNephrol. #&&&9#&:#6%-.

Item 34

Answer: D;alcium gluconateEducational Objective: Understand the mechanism o$ action o$ calcium salts when used totreat hyperalemia-induced cardiac to>icity.;alcium gluconate is least liely to reduce this patient,s serum potassium concentration. (heappropriate treatment o$ hyperalemia is primarily guided by its cardiac mani$estations. 3$ theelectrocardiogram shows hyperalemic cardiac to>icity! intravenous calcium salts are generallyindicated. ;alcium acts within minutes to reverse the cardiac e$$ects o$ hyperalemia.'owever! this is a direct membrane electrical e$$ect9 calcium salts do not reduce the serumpotassium concentration. (reatment that shi$ts potassium $rom the e>tracellular $luid into cellsshould also be started. 0uch treatment includes glucose and insulin! albuterol! and sodiumbicarbonate. 3$ total body potassium stores are increased! potassium must be removed via the

idneys! by dialysis! and*or via the gastrointestinal tract ?by administration o$ sodiumpolystyrene [email protected] A! Feidmann "! 0haw 0! <nadinger 4. E$$ect o$ various therapeutic approaches onplasma potassium and major regulating $actors in terminal renal $ailure. Am 5 4ed.69C:C&7-#.#.Allon 4. 'yperalemia in end-stage renal disease: mechanisms and management. 5 Am 0ocNephrol. 66C98: %-#.

Item 35

Page 19: Neurology Answers

8/11/2019 Neurology Answers

http://slidepdf.com/reader/full/neurology-answers 19/43

Answer: D(riple phosphate ?struvite@ stoneEducational Objective: =ecogni2e the clinical $eatures o$ struvite stone disease.0truvite ?in$ection@ stones occur most o$ten in women. "roteus species! a urea-splittingorganism! is $re)uently the cause o$ the chronic urinary tract in$ection that causes the stones."atients with medullary sponge idney and hypercalciuric stone $ormers have radiopa)uestones! but they will not have alaline urine! pyuria! or bacteriuria unless a superimposed

in$ection is present. Nephrolithiasis associated with renal tubular acidosis will cause an alalineurine and a low plasma bicarbonate concentration but will not cause pyuria or bacteriuria.ibliography.0chwart2 ! 0toller 4+. Nonsurgical management o$ in$ection-related renal calculi. Urol ;linNorth Am. 6669#8:78C-.#.=odman 50. 0truvite stones. Nephron. 6669:C&-6.

Item 36

Answer: Degin colchicine9 recommend subse)uent allopurinol! && mg daily! with a C&G to 7CGreduction o$ his a2athioprine doseEducational Objective: =ecogni2e the need $or special management o$ gout in renaltransplant recipients.=educed uric acid e>cretion can occur a$ter renal transplantation! leading to hyperuricemia and

occasional gouty arthritis. 'yperuricemia and gout are more liely to occur in patients who aretreated with cyclosporine. (he lower glomerular $iltration rate induced by cyclosporineprobably plays a role in uric acid retention! and renal tubular damage may also impair uric acide>cretion. <outy arthritis has been reported in 7G to #G o$ patients who are treated withcyclosporine but only rarely occurs in patients who are receiving a2athioprine alone.(he treatment o$ gout in renal transplant recipients has some ris. (he pre$erred therapy $orgout is colchicine in low doses! as higher doses may result in myoneuropathy due to decreasedhepatic clearance o$ colchicine mediated by cyclosporine ?or tacrolimus@. 3n addition tobeginning this patient on colchicine! his a2athioprine dose should be decreased and thesubse)uent dose o$ allopurinol should be appropriate $or his reduced glomerular $iltration rate.Nonsteroidal anti-in$lammatory drugs should be avoided in a patient with a2otemia because o$the ris o$ $urther reducing his impaired glomerular $iltration rate. Allopurinol should generallybe avoided in patients treated with a2athioprine! as allopurinol inter$eres with the metabolismo$ 8-mercapto-purine ?the major metabolite o$ a2athioprine@ and may produce severe bone

marrow to>icity. 3$ allopurinol is re)uired $or recurrent gout! a C&G to 7CG reduction in thedose o$ a2athioprine is recommended. A temporary increase in prednisone therapy may relievethe pain and in$lammation o$ active gout9 however! the concomitant %&&-mg dose o$allopurinol is too high $or this patient with a2otemia. Another alternative! which is not anoption in this )uestion! is changing the a2athioprine to mycophenolate! which does notinter$ere with allopurinol metabolism.ibliography.+in '! =ocher ++! 4cuillan 4A! 0chmalt2 0! "alella (D! o> 3'. ;yclosporine-inducedhyperuricemia and gout. N Engl 5 4ed. 669%#:#7-6#.#.Noord2ij (;! +eunissen 14! /an 'oo$$ 5". =enal handling o$ urate and the incidence o$ goutyarthritis during cyclosporine and diuretic use. (ransplantation. 669C#:8-7.%.urac DA! <ri$$ith "! (hompson 4E! 1ahl +E. 'yperuricemia and gout among hearttransplant recipients receiving cyclosporine. Am 5 4ed. 66#96#:-8..4inghelli <! 0eydou> ;! <oy 55! urnier 4. Uricosuric e$$ect o$ the angiotensin 33 receptor

antagonist losartan in heart transplant recipients. (ransplantation. 66988:#8-7.

Item 37

Page 20: Neurology Answers

8/11/2019 Neurology Answers

http://slidepdf.com/reader/full/neurology-answers 20/43

Answer: ?;@Aminoglycoside nephroto>icityEducational Objective: Evaluate and treat acute renal $ailure in a patient treated with anaminoglycoside antibiotic.(he time course and urinalysis are consistent with aminoglycoside nephroto>icity. "rerenala2otemia is unliely in the absence o$ signs o$ volume depletion or volume overload. (hespeci$ic gravity and high urinary sodium concentration are also inconsistent with prerenal

a2otemia. Although the patient was treated with ampicillin! which can be associated with acuteinterstitial nephritis! and the time course is plausible $or a diagnosis o$ acute interstitialnephritis! there is no evidence o$ rash or eosinophilia to mae this diagnosis more liely. 3naddition! the urinalysis! which does not show pyuria or low-grade proteinuria! would bedistinctly unusual in a patient with interstitial nephritis. (here is no evidence $orrhabdomyolysis in the absence o$ dipstic-positive heme or $or glomerulonephritis in theabsence o$ proteinuria! hematuria! or erythrocyte casts.Discontinuation o$ the aminoglycoside is indicated! and the prognosis $or improvement o$ renal$unction is good $or this patient. <lucocorticoids are not e$$ective $or treating aminoglycosidenephroto>icity and can be deleterious in a patient with in$ection. (he e$$icacy o$ dopamine hasnot been evaluated e>tensively in patients with aminoglycoside nephroto>icity! but dopaminehas not been shown to be e$$ective in patients with other $orms o$ acute renal $ailure. (here isno evidence that alalini2ation o$ the urine is bene$icial in treating aminoglycosidenephroto>icity.

ibliography.=yba 45! Abate 5! 1ang 0+! =u$$ing 45! +erner 0A! Drusano <+. "rospective evaluation o$the e$$ect o$ an aminoglycoside dosing regimen on rates o$ observed nephroto>icity andototo>icity. Antimicrob Agents ;hemother. 6669%:C6-CC.

Item 38

Answers: ?;@Discontinue gentamicinEducational Objective: Evaluate and treat acute renal $ailure in a patient treated with anaminoglycoside antibiotic.(he time course and urinalysis are consistent with aminoglycoside nephroto>icity. "rerenala2otemia is unliely in the absence o$ signs o$ volume depletion or volume overload. (hespeci$ic gravity and high urinary sodium concentration are also inconsistent with prerenala2otemia. Although the patient was treated with ampicillin! which can be associated with acute

interstitial nephritis! and the time course is plausible $or a diagnosis o$ acute interstitialnephritis! there is no evidence o$ rash or eosinophilia to mae this diagnosis more liely. 3naddition! the urinalysis! which does not show pyuria or low-grade proteinuria! would bedistinctly unusual in a patient with interstitial nephritis. (here is no evidence $orrhabdomyolysis in the absence o$ dipstic-positive heme or $or glomerulonephritis in theabsence o$ proteinuria! hematuria! or erythrocyte casts.Discontinuation o$ the aminoglycoside is indicated! and the prognosis $or improvement o$ renal$unction is good $or this patient. <lucocorticoids are not e$$ective $or treating aminoglycosidenephroto>icity and can be deleterious in a patient with in$ection. (he e$$icacy o$ dopamine hasnot been evaluated e>tensively in patients with aminoglycoside nephroto>icity! but dopaminehas not been shown to be e$$ective in patients with other $orms o$ acute renal $ailure. (here isno evidence that alalini2ation o$ the urine is bene$icial in treating aminoglycosidenephroto>icity.ibliography

.=yba 45! Abate 5! 1ang 0+! =u$$ing 45! +erner 0A! Drusano <+. "rospective evaluation o$the e$$ect o$ an amino-glycoside dosing regimen on rates o$ observed nephroto>icity andototo>icity. Antimicrob Agents ;hemother. 6669%:C6-CC.

Item 39

Page 21: Neurology Answers

8/11/2019 Neurology Answers

http://slidepdf.com/reader/full/neurology-answers 21/43

Answer: ;AD"1D is associated with hepatic cysts in up to C&G o$ patients as they ageEducational Objective: 1now some o$ the clinical $eatures o$ autosomal dominant polycysticidney disease.0ince! by de$inition! this disease has an autosomal dominant mode o$ inheritance! it occurswith e)ual $re)uency in both genders. 'owever! the disease does have varying clinicale>pression ?penetrance@! which e>plains why there is considerable variation in the onset and

progression o$ renal $ailure ?some patients do not develop end-stage renal disease until the8th or 7th decade o$ li$e@. oth blac and white populations are e)ually a$$ected. Autosomaldominant polycystic idney disease is the $ourth most common condition leading to end-stagerenal disease ?$ollowing hypertension! glomerulo-nephritis! and diabetes [email protected]."errone =D. E>trarenal mani$estations o$ AD"1D. 1idney 3nt. 6679C:#&##-%8.#.0essa A! <higgeri <4! (urco AE. Autosomal dominant polycystic idney disease: clinical andgenetic aspects. 5 Nephrol. 6679&:#6C-%&.

Item 40

Answer: ;3gA nephropathyEducational Objective: Understand the cause o$ glomerular proteinuria.(his patient has 3gA nephropathy. (he presence o$ erythrocyte casts de$ines the cause o$ the

dar urine and hematuria as glomerular in origin. 3gA nephropathy typically presents as darurine in a young person in association with an upper respiratory tract in$ection or participationin an athletic event. Although sicle cell disease may be associated with a $ocal segmentalpattern o$ glomerulosclerosis! this disorder does not cause episodes o$ dar urine. 'ematuria$rom medullary microin$arcts in patients with sicle cell disease is not o$ glomerular origin andnot associated with erythrocyte casts. oth poststreptococcal glomerulonephritis and lupusnephritis are typically associated with a low serum complement level.ibliography.<alla 5'. 3gA nephropathy. 1idney 3nt. 66C97:%77-7.#.Appel <. <lomerulonephritis. 3n <oldman +! ennett 5;! eds. ;ecil (e>tboo o$ 4edicine.#st ed. "hiladelphia: F 0aunders9 #&&&:C8-6. 3U: &&%%&8

Item 41

Answer: A

'ypertonic ?%G@ saline to increase the sodium concentration by & to C me)*+over severalhoursEducational Objective: =ecogni2e the importance o$ rapid correction o$ acute severehyponatremia that is associated with neuroto>icity."atients with schi2ophrenia o$ten develop symptomatic hyponatremia. (his may be due to thee$$ects o$ psychotropic drugs! increased thirst produced by dry mucous membranes! andperhaps a direct central nervous system mani$estation o$ schi2ophrenia itsel$. (hese patientssometimes ingest enormous )uantities o$ water over a very short period o$ time and candevelop pro$ound hyponatremia that causes neurologic dys$unction and sei2ures. 3$ sei2uresoccur! the syndrome o$ inappropriate antidiuretic hormone secretion may develop because asei2ure can cause the release o$ antidiuretic hormone. (his patient has pro$ound hyponatremiathat has probably caused a sei2ure. Fe can assume that his hyponatremia is o$ acute onset. 3nthis setting! the serum sodium concentration should be increased relatively rapidly toappro>imately #& to#C me)*+. (his can be accomplished most e$$ectively by in$using

hypertonic ?%G@ saline to increase the sodium concentration by & to C me)*+ over severalhours. Although physicians are appropriately concerned about the dangers o$ increasingsodium concentration too rapidly! this concern should be largely limited to patients withchronic hyponatremia. Under those circumstances! the serum sodium concentration should notbe increased by more than &.C me)*h or # me)*# h. "arenteral $urosemide ?to decreasema>imal renal concentrating mechanisms@ with strict water restriction may be use$ul in somepatients with 03AD' and chronic hyponatremia. 'owever! a more reasonable level o$ waterrestriction is about C&& to 8&& m+*# h. Normal saline is only indicated $or the treatment o$hyponatremia produced by volume depletion and is contraindicated in this patient.ibliography

Page 22: Neurology Answers

8/11/2019 Neurology Answers

http://slidepdf.com/reader/full/neurology-answers 22/43

.;heng 5;! Bios D! 0opici 'A! "eterson D=! isher 1A. +ong-term neurologic outcome inpsychogenic water driners with severe symptomatic hyponatremia: (he e$$ect o$ rapidcorrection. Am 5 4ed. 66&9:C8-8.#.erl (. (reating hyponatremia: damned i$ we do and damned i$ we don,t. 1idney 3nt.66&9%7:&&8-.%.1arp 3! +aureno =. "ontine and e>trapontine myelinolysis: a neurologic disorder $ollowingrapid correction o$ hyponatremia. 4edicine ?altimore@. 66%97#:%C6-7%.

.0terns ='. 0evere symptomatic hyponatremia: treatment and outcome. A study o$ 8 cases.Ann 3ntern 4ed. 679&7:8C8-8.

Item 42

Answer: D(his disorder appears to be related to -cell proli$eration induced by Epstein-arr virusin$ection in patients who are receiving chronic immunosuppressive therapyEducational Objective: =ecogni2e the prevalence! pathophysiology! and prominent clinical$eatures o$ posttransplant lymphoproli$erative disorders associated with chronicimmunosuppression in transplant recipients.(his patient has a posttransplant lymphoproli$erative disorder. +ymphoproli$erative disordersare among the most serious and potentially $atal complications o$ chronic immunosuppressionin transplant recipients. (hey are the most common posttransplant malignancy ?e>cluding sincancer and in situ cervical cancer@! accounting $or appro>imately #&G o$ all such

malignancies. (he prevalence is appro>imately G! which is %& to C& times higher than that inthe general population.+ymphoproli$erative disorders in transplant recipients have di$$erent characteristics $rom thosein the general population. (hey are mostly large cell lymphomas that are $irst detectedbecause o$ e>tranodal involvement. Appro>imately #&G to #CG o$ patients have centralnervous system involvement. (he neoplasms may also in$iltrate the renal allogra$t. Over 6&Go$ transplant recipients with posttransplant lymphoproli$erative disorders have non-'odgin,slymphomas! and the great majority o$ the non-'odgin,s lymphomas are o$ the -cell type.(he pathogenesis o$ posttransplant non-'odgin,s lymphoma appears to be related to -cellproli$eration induced by Epstein-arr virus in$ection in a patient who re)uires chronicimmunosuppression.(he ris is greatest in patients with more mared degrees o$ immunosuppression. (heincidence o$ posttransplant lymphoproli$erative disorders may be as high as G in hearttransplant recipients treated with high doses o$ muromonab ;D% ?O1(%@. (he use o$

prophylactic acyclovir or ganciclovir has been advocated to reduce the ris o$ posttransplantlymphoproli$erative disorders in patients treated with O1(%. (he degree o$ immunosuppressionis important in the development o$ posttransplant lymphoproli$erative disorders. (he incidenceo$ non-'odgin,s lymphoma is highest in the $irst year when immunosuppression is mostintense and is greater in heart transplant patients in whom a greater degree o$immunosuppression is re)uired. "osttransplant lymphoproli$erative disorders rarely cause'odgin,s lymphoma and are rarely o$ (-cell origin.ibliography.+iebowit2 D. Epstein-arr virus and a cellular signaling pathway in lymphomas $romimmunosuppressed patients. N Engl 5 4ed. 669%%:%-#.#."enn 3! "orat <. ;entral nervous system lymphomas in organ allogra$t recipients.(ransplantation. 66C9C6:#&-.%.'anto DF. ;lassi$ication o$ Epstein-arr virus-associated posttransplant lymphoproli$erativediseases: implications $or understanding their pathogenesis and developing rational treatment

strategies. Annu =ev 4ed. 66C98:%-6..0winnen +5! ;ostan2o-Nordin 4=! isher 0<! O,0ullivan E5! 5ohnson 4=! 'erou> A+! et al.3ncreased incidence o$ lymphoproli$erative disorder a$ter immunosuppression with themonoclonal antibody O1(% in cardiac-transplant recipients. N Engl 5 4ed. 66&9%#%:7#%-.C.O,rien 0! ernert =A! +ogan 5+! +ien '. =emission o$ posttransplant lymphoproli$erativedisorder a$ter inter$eron al$a therapy. 5 Am 0oc Nephrol. 6679:%-6.

Item 43

Page 23: Neurology Answers

8/11/2019 Neurology Answers

http://slidepdf.com/reader/full/neurology-answers 23/43

Answer: DDiscontinue immune globulin <Educational Objective: 1now how to manage acute renal $ailure in a patient treated withintravenous immune globulin <.3mmune globulin < is made $rom pooled human plasma stabili2ed with glucose! maltose!glycine! sucrose! sorbitol! or albumin. Although intravenous immune globulin has beenlicensed by the ood and Drug Administration ?DA@ $or the treatment o$ primary

immunode$iciencies! immune-mediated thrombocytopenia! 1awasai,s syndrome! chronic -cell lymphoblastic leuemia! and pediatric '3/ in$ection and $or use in bone marrow transplantrecipients #& years o$ age or younger! it has also been used in the treatment o$ variousneurologic and hematologic diseases. 4ore than #& cases o$ renal insu$$iciency associatedwith its use have been reported to the DA. (he incidence o$ this complication is not nown!but it appears to be uncommon.(he pathogenesis o$ the renal disease associated with administration o$ immune globulin isunnown. Although an e$$ect o$ the immune globulin is biologically plausible! someinvestigators have implicated the stabili2ing agents. 4ost cases occur within wee o$initiation o$ therapy. "ree>isting renal disease and increasing age appear to be ris $actors! butthe e$$ects o$ concomitant nephroto>ins have not been evaluated. (he median age o$ patientswith this complication reported to the DA is 8&.C years9 &G o$ patients re)uire dialysis. (hemean time $or recovery o$ renal $unction is & days a$ter discontinuation o$ therapy. Althoughmost patients recovered renal $unction! mortality was CG! presumably because o$ the

severity o$ the underlying disease. (he most $re)uent $inding on renal biopsy is vacuoli2ationo$ pro>imal tubular epithelial cells. 0uccess$ul therapeutic strategies a$ter $ull recovery o$ renal$unction in patients in whom intravenous immune globulin is necessary include diluting theimmune globulin and decreasing the dose and rate o$ administration.3mmune globulin therapy should be discontinued in this patient. 'er physical e>aminationsuggests that she is volume overloaded. (here$ore! administration o$ intravenous $luids isinappropriate and may worsen cardiac hemodynamics i$ she is oliguric. Although somepatients may re)uire dialysis! the need is usually transient! and preparation $or end-stagerenal disease is generally not indicated. Although plasmapheresis has been used in thetreatment o$ chronic in$lammatory demyelinating polyneuropathy! its role in treating immuneglobulin-induced acute renal $ailure has not been studied.ibliography. Dalaas 4;. 3ntravenous immune globulin therapy $or neurologic diseases. Ann 3ntern 4ed.6679#8:7#-%&.

#.=enal insu$$iciency and $ailure associated with immune globulin intravenous therapyJUnited0tates! 6C-66. 44F= 4orb 4ortal Fly =ep. 6669:C-#.

Item 44

Answer: ;4etabolic acidosis and metabolic alalosisEducational Objective: =ecogni2e that when a large anion gap coe>ists with a normal serumbicarbonate concentration a mi>ed metabolic acidosis and metabolic alalosis may be present.(his patient has metabolic acidosis and metabolic alalosis. 'e has progressive and severerenal $ailure and would be e>pected to have an anion gap metabolic acidosis. 3ndeed! his aniongap is % me)*+. (his suggests that his anion gap has increased by about 6 me)*+ ?$rom thenormal range o$ about # me)*+@. 'is bicarbonate concentration there$ore should be reducedby about 6 me)*+. 3nstead! it is within the normal range at # me)*+. 'is arterial blood gasresults are also within the normal range. (he patient also has a history o$ nausea and

vomiting! which have probably produced a metabolic alalosis. (his has raised his serumbicarbonate concentration $rom about 8 me)*+ bac to # me)*+ ?and simultaneously reducedhis serum chloride concentration@. (his picture is most compatible with anion gap metabolicacidosis due to uremia and metabolic alalosis due to vomiting. (he patient certainly has ami>ed acid-base disturbance despite his normal arterial blood p'! "co#! and bicarbonateconcentration. 'e has no evidence o$ respiratory alalosis ?the "co# is normal@. Althoughrespiratory alalosis and respiratory acidosis could conceivably coe>ist and neutrali2e oneanother! this particular mi>ed disorder is rarely! i$ ever! diagnosed clinically and would notaccount $or an increased anion gap.ibliography

Page 24: Neurology Answers

8/11/2019 Neurology Answers

http://slidepdf.com/reader/full/neurology-answers 24/43

.Emmett 4! Narins =<. ;linical use o$ the anion gap. 4edicine. 6779C8:%-C.#.Narins =<! Emmett 4. 0imple and mi>ed acid-base disorders: a practical approach.4edicine. 6&9C6:8-7.

Item 45

Answer: 0arcoidosis

Educational Objective: 3denti$y hypercalcemia as a cause o$ renal impairment and correctlyidenti$y sarcoidosis as the most liely cause o$ the hypercalcemia.(his patient has mild pree>isting a2otemia with additional signi$icant impairment o$ renal$unction associated with severe hypercalcemia. 'ypercalcemia may be associated with manynonspeci$ic symptoms related to gastrointestinal! cardiovascular! neuromuscular! andneuropsychiatric disturbances. =enal mani$estations may include nephrolithiasis associatedwith hypercalciuria! nephrogenic diabetes insipidus associated with polyuria! distal renaltubular acidosis! and renal insu$$iciency. 0erum calcium values o$ # to C mg*d+ may cause areversible $all in the glomerular $iltration rate that is mediated by direct renal vasoconstrictionand natriuresis-induced volume depletion! as occurred in this patient. Although there are morethan #& causes o$ hypercalcemia! primary hyperparathyroidism or malignancy usuallyaccounts $or &G to 6&G o$ cases. (he elevated serum calcitriol level ?!#C-dihydro>yvitaminD@ suggests that the hypercalcemia is liely due to a granulomatous disease such assarcoidosis. ecause this patient,s serum parathyroid hormone value is low-normal during a

severe hypercalcemic phase! he is unliely to have hyperparathyroidism. Normal serum andurine immunoglobulin values without a monoclonal m-spie mae multiple myeloma veryunliely. 0imilarly! the clinical presentation and increased !#C-dihydro>y-vitamin D level maemetastatic lung cancer unliely. 0ubse)uent testing o$ this patient e>cluded the diagnosis o$humoral hypercalcemia o$ malignancy because no serum parathyroid hormoneJrelated proteinwas detected. inally! the diagnosis o$ Fegener,s granulomatosis is e>cluded because there isno evidence o$ glomerulonephritis based upon the patient,s urinalysis.ibliography.;asella 5! Allon 4. (he idney in sarcoidosis. 5 Am 0oc Nephrol. 66%9%:CCC-8#.#.Beimer '5! <reenaway (4! 0lavin 5! 'ards D1! Bhou '! Doery 5;! et al. "arathyroid-hormone-related protein in sarcoidosis. Am 5 "athol. 669C#:7-#.%.Finnacer 5+! ecer 1+! 1at2 0. Endocrine aspects o$ sarcoidosis. N Engl 5 4ed.689#7:#7-%.

Item 46

Answer: A"rednisoneEducational Objective: Understand the pathophysiology and appropriate therapy $orhypercalcemia associated with sarcoidosis.(he paratracheal lymph node biopsy specimen showing $oci o$ histiocytes and giant cells ?thatis! noncaseating granulomas@ with negative stains $or acid-$ast bacilli and $ungi con$irms thediagnosis o$ sarcoidosis as the cause o$ this patient,s hypercalcemia. =enal disease in patientswith sarcoidosis may include hypercalciuria*hypercalcemia ?nephrolithiasis or impairedglomerular $iltration rate@! granulomatous interstitial nephritis! rare glomerular diseasesmani$ested by heavy proteinuria or erythrocyte casts! or rare urinary tract obstructionsecondary to enlarged retroperitoneal lymph nodes or $ibrosis ecause o$ his rapidimprovement in renal $unction with the correction o$ his hypercalcemia! this patient,s renalimpairment is most liely directly related to his hypercalcemia. 'yperabsorption o$ dietary

calcium is )uite common in patients with sarcoidosis! leading to hypercalciuria in %&G to C&Go$ patients and hypercalcemia in CG to #&G o$ patients. (he primary de$ect in patients withsarcoidosis and other granulomatous disorders is increased e>trarenal production o$ calcitriol?!#C-dihydro>yvitamin DMthe most active metabolite o$ vitamin D@ by activated mononuclearcells! especially macrophages! in the lungs and lymph nodes.(herapy with low-dose glucocorticoids! & to %& mg o$ prednisone daily! will inhibit calcitriolsynthesis by the activated mononuclear cells. (he data do not support the diagnosis o$multiple myeloma or any other malignancy so that the more aggressive chemotherapy listed inthe other options is not indicated.ibliography

Page 25: Neurology Answers

8/11/2019 Neurology Answers

http://slidepdf.com/reader/full/neurology-answers 25/43

.Adams 50. /itamin D metabolite-mediated hypercalcemia. Endocrinol 4etab ;lin North Am.669:78C-7.#.Adams 50! =en 0. Autoregulation o$ !#C-dihydro>yvitamin D synthesis in macrophagemitochondria by nitric o>ide. Endocrinology. 6689%7:C-7.%.Beimer '5! <reenaway (4! 0lavin 5! 'ards D1! Bhou '! Doery 5;! et al. "arathyroid-hormone-related protein in sarcoidosis. Am 5 "athol. 669C#:7-#.

Item 47

Answer: egin dialysisEducational Objective: 1now how to treat a patient with oliguric acute renal $ailure.(his oliguric patient who is resistant to diuretics has hyperalemia and an anion gap metabolicacidosis with acute tubular necrosis caused by several $actors ?sepsis! hypotension! contrastadministration! and e>posure to nephroto>ic drugs@. "hysical e>amination is consistent withvolume overload. (he patient there$ore has several indications $or acute renal replacementtherapy. (here is much controversy about whether the type o$ membrane used $or dialysis o$patients with acute renal $ailure is independently associated with outcome. 0o-calledbiocompatible membranes are not associated with complement activation! whereasbioincompatible membranes o$ten result in activation o$ complement and can be associatedwith an anaphylactic-lie response.0everal previous controlled clinical trials have suggested that treatment with biocompatible

membranes resulted in a better outcome in patients with acute renal $ailure. A recent study by5orres et al. compared patients treated with polymethyl-methacrylate dialy2ers representingbiocompatible membranes with patients treated with cuprophane membranes in thebioincompatible arm. 3n contrast to earlier studies! there was no di$$erence in survival daysa$ter beginning treatment in patients treated with either membrane. (he reasons $or thesedisparities are unnown! but neutrophil activation by dialysis membranes may be a moreimportant determinant o$ deleterious e$$ects than activation o$ the complement cascade.Despite this study! the role o$ treatment with di$$erent membranes has not been conclusivelydetermined. Nonetheless! this patient re)uires dialysis as a li$esaving -measure.(he need $or dialysis taes precedence over obtaining imaging studies or observation a$terdiscontinuing one o$ the agents implicated in the pathogenesis o$ the acute renal $ailure.Aminoglycoside therapy is liely to be only one o$ many causes o$ renal $ailure in this patient.Although discontinuing the aminoglycoside may be indicated! this is not liely to have an e$$ecton the course o$ the acute complications. 4ost patients with acute tubular necrosis do not

undergo renal biopsy because the diagnosis may be made reliably on clinical grounds! andbiopsy $indings in patients with acute tubular necrosis rarely dictate changes in therapy.ibliography.5orres A! <ahl <4! Dobis ;! "olenaovic 4'! ;aalarosi 1! =utowsi ! et al.'emodialysis-membrane biocompatibility and mortality o$ patients with dialysis-dependentacute renal $ailure: a prospective randomised multicentre trial. 3nternational 4ulticentre 0tudy<roup. +ancet. 6669%C:%%7-.#.'immel$arb 5! 'aim =4. (he use o$ biocompatible dialysis membranes in acute renal$ailure. Adv =en =eplace (her. 6679 ?# 0uppl @:7#-&.%.'immel$arb 5! (olo$$ =ubin N! ;handran "! "arer =A! Fingard =+! 'aim =. A multicentercomparison o$ dialysis membranes in the treatment o$ acute renal $ailure re)uiring dialysis. 5Am 0oc Nephrol. 6696:#C7-88.

Item 48

Answer: AEnhanced renal sodium! water! and urea reabsorption as a result o$ volume-mediatedstimulation o$ anti-diuretic hormone and renin-angiotensin-aldosteroneEducational Objective: =ecogni2e the reasons $or an elevated blood urea nitrogen:serumcreatinine ratio in patients with e>tracellular $luid volume depletion.'igh $ever ?increased catabolic state@! gastrointestinal bleeding! and treatment with high-doseglucocorticoids may all cause increased production o$ urea and thus elevate the blood ureanitrogen:serum creatinine ratio. 'owever! the most common reason $or an elevated ratio isreduced renal per$usion caused by e>tracellular $luid volume depletion! as occurred in thispatient. Other causes $or impaired renal per$usion and an increased blood urea nitrogen:serum

Page 26: Neurology Answers

8/11/2019 Neurology Answers

http://slidepdf.com/reader/full/neurology-answers 26/43

creatinine ratio include cardiac $ailure or cirrhosis in which there is reduced e$$ective circulatingvolume ?arterial under$illing@. 0ince urea is a small molecule and $reely e)uilibrates across cellmembranes! its elevation could only be due to increased production or diminished e>cretionbut would not be due to shi$ts.ibliography.Dossetor 5. ;reatininemia versus uremia: (he relative signi$icance o$ blood urea nitrogenand serum creatinine concentrations in a2otemia. Ann 3ntern 4ed. 68898C:#7-66.

Item 49

Answer: DDi$$use proli$erative lupus nephritisEducational Objective: =ecogni2e the clinical $eatures associated with the various histologicpatterns o$ lupus nephritis.(his patient with multiple $eatures o$ systemic lupus erythematosus has abnormal urinesediment! major renal dys$unction with elevated blood urea nitrogen and serum creatininelevels! hypertension! and abnormal serologic studies with a positive anti-DNA antibody and alow serum complement level. All point to a diagnosis o$ di$$use proli$erative lupus nephritis."atients with membranous lupus nephropathy rarely have such active serologic $indingsdespite the common $inding o$ heavy proteinuria. "atients with mesangial and $ocalproli$erative lupus nephritis have less proteinuria and less active serologic $indings.=ecogni2ing which clinical $eatures are associated with the various histologic patterns o$ lupus

nephritis is important! since this helps de$ine which patients re)uire a renal biopsy ?mostpatients with di$$use proli$erative disease but very $ew with mesangial disease@. 3t also helpsde$ine the type o$ therapy. /igorous therapy is needed $or patients with more severeproli$erative involvement! and less aggressive treatment is appropriate $or patients withmembranous and the milder proli$erative disorders.Other important prognostic $actors $or patients with lupus nephritis have recently beenclari$ied. actors present at the time o$ renal biopsy that predict a poor long-term renaloutcome and progression to renal $ailure include blac race! elevated serum creatinine level!heavier proteinuria! severe anemia! and the histologic $indings o$ combined severe activity andchronicity as well as both crescents and tubulointerstitial $ibrosis on renal biopsy specimens.ibliography.Appel <. <lomerulonephritis. 3n: <oldman +! ennett 5;! eds. ;ecil (e>tboo o$ 4edicine.#st ed. "hiladelphia: F 0aunders: #&&&:C8-6. "ub4ed +in8#.Appel <! =adharishnan 5D! Agati /. 0econdary glomerular diseases. 3n: renner 4!

=ector ;! eds. (he 1idney. 8th ed. "hiladelphia: F 0aunders9 #&&&. "ub4ed +in6%.Appel <! D,Agati /. =enal involvement in systemic lupus erythematosus. 3n: 4assry 0<!<lassoc =5. 4assry Q <lassoc,s (e>tboo o$ Nephrology. th ed. "hiladelphia: +ippincottFilliams Q Filins9 #&&&. "ub4ed +in.Austin 'A %rd! oumpas D(! /aughan E4! alow 5E. "redicting renal outcomes in severelupus nephritis: contributions o$ clinical and histologic data. 1idney 3nt. 669C:C-C&.

Item 50

Answer: ;4onthly pulse intravenous cyclophosphamide plus oral glucocorticoidsEducational Objective: Understand the treatment o$ patients with severe lupus nephritis.3n controlled clinical trials! intravenous cyclophosphamide combined with very-low-dose oralglucocorticoids has been shown to be more e$$ective in preventing renal $ailure than oral orintravenous glucocorticoids alone. Oral cyclo-sporine is used along with other $orms o$ therapy

but is not a $irst-line medication $or patients with di$$use proli$erative lupus nephritis.ibliography.oumpas D(! Austin 'A %rd! /aughn E4! 1lippel 5'! 0teinberg AD! arboro ;'! et al.;ontrolled trial o$ pulse methylprednisolone versus two regimens o$ pulse cyclophosphamide insevere lupus nephritis. +ancet. 66#9%&:7-C!#.<ourley 4! Austin 'A %rd! 0cott D! arboro ;'! /aughn E4! 4uir 5! et al.4ethylprednisolone and cyclophosphamide! alone or in combination! in patients with lupusnephritis. A randomi2ed! controlled trial. Ann 3ntern 4ed. 6689#C:C6-C7. U3: 68%6%.Appel <! =adharishnan 5D! Agati /. 0econdary glomerular diseases. 3n: renner 4!=ector ;! eds. (he 1idney. 8th ed. "hiladelphia: F 0aunders9 #&&&. "ub4ed +in6

Page 27: Neurology Answers

8/11/2019 Neurology Answers

http://slidepdf.com/reader/full/neurology-answers 27/43

.Appel <! D,Agati /. =enal involvement in systemic lupus erythematosus. 3n: 4assry 0<!<lassoc =5. 4assry Q <lassoc,s (e>tboo o$ Nephrology. th ed. "hiladelphia: +ippincottFilliams Q Filins9 #&&&. "ub4ed +in

Item 51

Answer: ;A %C-year-old man who has ingested a large amount o$ aspirin in a suicide attempt

Educational Objective: =ecogni2e coe>isting anion gap metabolic acidosis and respiratoryalalosis and now that this mi>ed disorder should suggest the possibility o$ salicylateinto>ication.(he laboratory studies show a low bicarbonate concentration and an increased anion gap ?#%me)*+@ consistent with anion gap metabolic acidosis. Arterial blood gas studies show analaline p' with a very low "co# o$ # mm 'g. (his indicates coe>isting respiratory alalosis.(he combination o$ respiratory alalosis and anion gap metabolic acidosis occurs commonly inolder children and adults with salicylate poisoning! such as the %C-year-old man whoattempted suicide. oung children who are poisoned by salicylates more $re)uently presentwith metabolic acidosis and a low p' because they are less liely to develop respiratoryalalosis. Adults with salicylate poisoning may not develop respiratory alalosis i$ theysimultaneously ingest other drugs that are respiratory depressants! such as barbiturates."atients with surreptitious la>ative abuse may develop hyperchloremic metabolic acidosis?$rom the loss o$ sodium bicarbonate in the stool@ or! less commonly! metabolic alalosis

associated with severe hypoalemia. ulimia and vomiting cause a metabolic alalosis.ibliography.<abow "A! Anderson =5! "otts DE! 0chrier =F. Acid-base disturbances in the salicylate-into>icated adult. Arch 3ntern 4ed. 679%:-.#.ailey =! 5ones 0=. ;hronic salicylate into>ication. A common cause o$ morbidity in theelderly. 5 Am <eriatr 0oc. 669 %7:CC8-8.

Item 52

Answer: egin an angiotensin-converting en2yme inhibitorEducational Objective: =ecogni2e erythrocytosis as a complication in a renal transplantrecipient and now the appropriate therapy."osttransplant erythrocytosis ?"(E@! de$ined as a hematocrit T CG! a$$ects &G to CG o$renal transplant recipients and $re)uently develops in patients with e>cellent allogra$t $unction.

"(E may be stimulated either by e>cessive erythropoietin release $rom the native idney or byother unnown mechanisms since erythropoietin levels may occasionally be normal orsuppressed. (hese latter mechanisms ?perhaps cytoines or angiotensin 33@ may eitherenhance sensitivity to erythropoietin or directly promote erythropoiesis."(E can be e$$ectively treated with angiotensin-converting en2yme ?A;E@ inhibitors ortheophylline. +ow-dose A;E inhibitors can reduce the hematocrit to near normal levels inalmost all patients with "(E. (he e$$ect usually begins within 8 wees o$ initiating therapy andis complete in % to 8 months. Use o$ A;E inhibitors may induce a mild anemia in any renaltransplant recipient and may increase erythropoietin re)uirements in dialysis patients. Anassociation between A;E inhibitor-induced decrease in hematocrit and decline in erythropoietinlevels has been demonstrated in some patients in whom erythropoietin levels were initiallynormal or elevated. 'owever! the association is not always noted because A;E inhibitors mayalso reduce the hematocrit in transplant recipients with "(E whose initial erythropoietin levelsare -suppressed.

(he mechanism by which A;E inhibitor therapy corrects the hematocrit in patients with "(Eremains uncertain. Direct activation o$ the angiotensin 33 receptor antagonist appears to beone possibility because "(E can be e$$ectively treated with losartan! an angiotensin 33 receptor?A( blocer@. (he mechanism by which activation o$ the A( receptor leads to erythrocytosisis unnown! but such activation may diminish erythropoietin production.A2athioprine and cyclosporine have not been causally associated with "(E. (here$ore!changing these drugs is not indicated. 4elphalan is also not indicated because chemotherapywith this drug or with other agents has not been shown to be e$$ective in patients with "(E.

Page 28: Neurology Answers

8/11/2019 Neurology Answers

http://slidepdf.com/reader/full/neurology-answers 28/43

ibliography.<aston =0! 5ulian A! ;urtis 55. "osttransplant erythrocytosis: an enigma revisited. Am 51idney Dis. 669#:-.#."era2ella 4! 4c"hedran "! 1liger A! +orber 4! +evy E! ia 45. Enalapril treatment o$posttransplant erythrocytosis: -e$$icacy independent o$ circulating erythropoietin levels. Am 51idney Dis. 66C9#8:6C-C&&.%.<ossmann 5! (hurmann "! achmann (! Feller 0! 1achel '<! 0choeppe F! et al. 4echanism

o$ angiotensin-converting en2yme inhibitor-related anemia in renal transplant recipients.1idney 3nt. 6689C&:67%-..1laassen =5! van <elder (! =ischen-/os 5! Deinum 5! 4an in,t /eld A5! Feimar F. +osartan!an angiotensin-33 receptor antagonist! reduces hematocrits in idney transplant recipients withposttransplant erythrocytosis. (ransplantation. 66798:7&-#.

Item 53

Answer: AObtain ;( o$ the idneys and pelvis without contrastEducational Objective: =ecogni2e retroperitoneal $ibrosis as a potential and treatable cause o$ obstructive uropathy.;ommunity-ac)uired acute renal $ailure is relatively rare. 4ost patients with community-ac)uired acute renal $ailure on hospital admission have prerenal a2otemia or a reducedglomerular $iltration rate secondary to medications! o$ten nonsteroidal anti-in$lammatory

drugs.(he diagnosis o$ chronic renal insu$$iciency must be considered in this patient! but there is nohistory o$ the two most common diseases associated with this entity! diabetes mellitus andhypertension. (he lielihood o$ chronic renal $ailure is also reduced by the $inding o$ normal-si2ed idneys. (his patient,s medical history is intriguing! especially because o$ the blandurinalysis.(he approach to a2otemia re)uires ruling out prerenal and obstructive causes be$oreconsidering intrinsic renal diseases. (he development o$ retroperitoneal $ibrosis a$terabdominal surgery and pelvic bleeding or in association with pelvic malignancies is well nown.3n addition! patients with retroperitoneal $ibrosis o$ten do not have hydronephrosis onultrasound e>amination. (he recovery o$ renal $unction in patients with urinary tractobstruction is o$ten predicated on the duration o$ high-grade obstruction! with a long courseassociated with poor response o$ the glomerular $iltration rate."hysical e>amination showed that this patient is volume overloaded! and there$ore volume

repletion is contraindicated. Although patients with volume disorders o$ten have prerenala2otemia! the high urinary sodium concentration is not consistent with this diagnosis.'owever! the microscopic e>amination does not suggest glomerular! interstitial! renal tubular!or venous disease. Although the presentation is consistent with renal arterial disease! the highurinary sodium concentration and the absence o$ an underlying condition predisposing tobilateral renal vascular disease mae this diagnosis less liely. Use o$ contrast agents in such asituation must be care$ully considered. ;ontrast agents should be used in patients with renalinsu$$iciency only i$ there is a high e>pectation o$ a positive $inding that is amenable totherapy.Enrolling a patient in a chronic end-stage renal disease program without complete evaluationto rule out obstruction would condemn the patient to a li$e o$ dialysis despite a potentiallyreversible disease.(his patient was $ound to have an abnormal pelvic ;( scan! consistent with retroperitoneal$ibrosis. (he insertion o$ a ureteral stent resulted in diuresis and a $all in his serum creatinine

concentration to .7 mg*d+ over wee. 'e later underwent success$ul bilateral ureterolysis.ibliography.1au$man 5! Dhaal 4! "atel ! 'amburger =. ;ommunity-ac)uired acute renal $ailure. Am 51idney Dis. 6697:6-.#.1oellier 0+! ;ronan 55. Acute urinary tract obstruction. 3maging update. Urol ;lin North Am.6679#:C7-#.%.1eith D0! +arson (0. 3diopathic retroperitoneal $ibrosis. 5 Am 0oc Nephrol. 66%9%:7-C#..4artine2-4aldonado 4! 1umjian DA. Acute renal $ailure due to urinary tract obstruction. 4ed;lin North Am. 66&9 7:66-%#.Item 54

Page 29: Neurology Answers

8/11/2019 Neurology Answers

http://slidepdf.com/reader/full/neurology-answers 29/43

Answer: ;;hange $urosemide to aceta2olamideEducational Objective: Understand that aceta-2olamide is a diuretic that causesbicarbonaturia and there$ore leads to metabolic acidosis! unlie most other diuretics thatcause metabolic alalosis. (his property may be very use$ul in certain patients such as thepatient described here."atients with chronic obstructive pulmonary disease o$ten develop chronic respiratory acidosis

mani$ested by an increased arterial blood "co# and a compensatory increase in thebicarbonate concentration. A chronic elevation o$ the arterial blood "co# increases renal acide>cretion and also increases the renal threshold $or bicarbonate e>cretion. (he bicarbonateconcentration will increase by appro>imately &. me) $or each -mm 'g increase in arterialblood "co# above & mm 'g. (his degree o$ compensation results in a slightly acidic arterialblood p' ?less than 7.%8@. "atients with chronic respiratory acidosis who are treated with loopor thia2ide diuretics may develop metabolic alalosis. (he additional increase in bicarbonateconcentration will raise the p' to normal or even into the alaline range! which has occurred inthis patient. (he p' o$ 7. and bicarbonate concentration o$ %8 me)*+ in a patient withnown chronic obstructive pulmonary disease and a high arterial blood "co# are mostconsistent with chronic respiratory acidosis and superimposed metabolic alalosis ?probablydue to the loop diuretics@. Development o$ metabolic alalosis in such patients can reducerespiratory drive and mae weaning $rom a ventilator more di$$icult. 3t is there$ore importantto reduce their bicarbonate concentration to the appropriate compensatory level. (his can be

accomplished by volume e>pansion ?when the patient is volume depleted@ or by switching toan acidi$ying diuretic such as aceta2olamide ?when the patient re)uires $urther diuresis@. 0incethis patient continues to have cracles and edema! continued diuresis is appropriate! and achange $rom $urosemide to aceta2olamide is indicated.;ontinuing aggressive diuresis with loop diuretics will $urther e>acerbate his metabolicalalosis. Administration o$ saline is contraindicated because o$ the evidence o$ persistentvolume e>cess. Arteriovenous ultra$iltration would only be indicated i$ he developed salt and$luid retention that was re$ractory to all diuretics and i$ he had more severe renal insu$$iciency.ibliography.0atrud 5! Dempsey 5A. =elative e$$ectiveness o$ aceta2olamide versusmedro>yprogesterone acetate in correction o$ chronic carbon dio>ide retention. Am =ev =espirDis. 6%9#7: &C-#.#.4iller "D! erns A0. Acute metabolic alalosis perpetuating hypercarbia. A role $oraceta2olamide in chronic obstructive pulmonary disease. 5A4A. 677:#%9#&&-.

Item 55

Answer: D"atients with white coat hypertension have a higher systemic vascular resistance and le$tventricular mass inde> than their normotensive counterparts and may have an increased riso$ cardiovascular diseaseEducational Objective: =ecogni2e the importance o$ Rwhite coatS hypertension and its riss$or cardiovascular disease."atients with Rwhite coatS hypertension may have subtle $unctional abnormalities whencompared with their normotensive counterparts. Abnormalities include a higher systemicvascular resistance! higher insulin and triglyceride levels! a higher le$t ventricular mass inde>!and increased carotid artery intimal medial thicness. (hese patients are also more liely todevelop sustained hypertension. Overall! patients with Rwhite coatS hypertension may have anincreased ris $or cardiovascular disease! although the ris o$ cardiovascular complications

appears to be relatively low i$ the ambulatory blood pressure remains normal. RFhite coatS hypertension may be de$ined as the presence o$ o$$ice hypertension when thepatient,s blood pressure is repeatedly normal when measured by the patient or by others athome! at wor! or by #-hour ambulatory blood pressure monitoring. 'owever! #-hourambulatory blood pressure monitoring is not re)uired $or the diagnosis. (he ris o$hypertensive cardiovascular complications ?including both the development and regression o$le$t ventricular hypertrophy@ correlates more closely with #-hour or daytime ambulatoryblood pressure monitoring than with o$$ice pressure readings. RFhite coatS hypertension maya$$ect up to #&G o$ patients with mild hypertension. One way to minimi2e the Rwhite coatSe$$ect is to have blood pressure in the o$$ice taen by a nurse rather than by the physician. A

Page 30: Neurology Answers

8/11/2019 Neurology Answers

http://slidepdf.com/reader/full/neurology-answers 30/43

study using continuous interarterial monitoring has documented a more pronounced increasein systolic blood pressure ?V ## mm 'g@ when blood pressure was measured with a sphygmo-manometer by an un$amiliar physician rather than by a nurse.ibliography."icering (<! 5ames <D! oddie ;! 'arsh$ield <A! lan 0! +aragh 5'. 'ow common is whitecoat hypertensionH 5A4A. 69#C6:##C-.#.4ancia <! "arati <! "omidossi <! <rassi <! ;asadei =! Banchetti A. Alerting reaction and rise

in blood pressure during measurement by physician and nurse. 'ypertension. 679 6:#&6-C.%."alatini "! 4ormino "! 0antonastaso 4! 4os +! Dal ollo 4! Banata <! et al. (arget-organdamage in stage 3 hypertensive subjects with white coat and sustained hypertension: results$rom the 'A=/E0( study. 'ypertension. 669%:C7-8%..5ulius 0! 4ejia A! 5ones 1! 1rause +! 0chor N! van de /en ;! et al. RFhite coatS versus RsustainedS borderline hypertension in (ecumseh! 4ichigan. 'ypertension. 66&98:87-#%.C.;avallini 4;! =oman 45! "icering (<! 0chwart2 5E! "ini =! Devereu> =. 3s white coathypertension associated with arterial disease or le$t ventricular hypertrophyH 'ypertension.66C9#8:%-6.

Item 56

Answer: D0he should be told that better control o$ blood pressure will diminish her ris o$ developingpreeclampsia

Educational Objective: 1now how to treat and counsel a pregnant patient with pree>istinghypertension."atients with pree>isting hypertension who become pregnant present a number o$ potentialproblems that should be $ollowed closely. (his patient currently has hypertension at the end o$the $irst trimester o$ pregnancy. Angiotensin-converting en2yme inhibitors ?and perhapsangiotensin 33 receptor blocers@ should be discontinued because women who tae these drugsin the last two trimesters o$ pregnancy have a ris o$ $etal complications such asoligohydramnios! renal and maturational $ailure! and $etal death. 'ypertension should betreated because there is a lower ris o$ $etal and maternal morbidity and mortality in pregnantwomen whose blood pressure is lower than #&*7C mm 'g. "atients with chronic hypertensionhave an appro>imately #CG increased ris o$ developing preeclampsia and should be $ollowedcare$ully $or the development o$ proteinuria. "atients with a dia-stolic blood pressure greaterthan && mm 'g have a higher ris o$ developing preeclampsia than those with a lower dia-stolic blood pressure. Un$ortunately! treatment o$ hypertension does not decrease the ris o$

developing preeclampsia.ibliography.0ibai 4! +indheimer 4! 'auth 5! ;aritis 0! /anDorsten "! 1lebano$$ 4! et al. =is $actors $orpreeclampsia! abruptio placentae! and adverse neonatal outcomes among women with chronichypertension. National 3nstitute o$ ;hild 'ealth and 'uman Development Networ o$ 4aternal-etal 4edicine Units. N Engl 5 4ed. 669%%6:887-7.

Item 57

Answer: ;'ydrochlorothia2ideEducational Objective: 1now the treatment to reduce urinary calcium e>cretion.'ydrochlorothia2ide reduces urinary calcium e>cretion by enhancing distal calciumreabsorption directly and by producing mild e>tracellular $luid volume contraction that causesenhanced sodium and calcium reabsorption in the pro>imal and loop segments o$ the nephron.

Dietary calcium restriction has been shown to result in negative calcium balance in patientswith hypercalciuric stone disease and is an ine$$ective way to reduce urinary calcium e>cretion.urosemide inhibits sodium and calcium reabsorption in the loop o$ 'enle and thus wouldincrease urinary calcium e>cretion. ;ranberry juice and high $luid intae would have no e$$ecton urinary calcium e>cretion.ibliography.ushinsy DA. Nephrolithiasis. 5 Am 0oc Nephrol. 669 6:67-#.

Item 58

Answer: D

Page 31: Neurology Answers

8/11/2019 Neurology Answers

http://slidepdf.com/reader/full/neurology-answers 31/43

Nephronophthisis and medullary cystic diseaseEducational Objective: =ecogni2e the inherited disorders that are associated with renal$ailure.Nephronophthisis and medullary cystic disease usually cause end-stage renal disease inadolescents or young adults. ;linical $eatures include polyuria! thirst! and renal salt wasting!but hypertension does not usually develop. None o$ the other disorders are usually associatedwith renal $ailure. 4edullary sponge idney is associated with microhematuria and renal

stones! and horseshoe idney is associated with renal stones. 3nherited $orms o$ nephrogenicdiabetes insipidus are usually diagnosed in in$ancy! and benign $amilial hematuria is associatedonly with hematuria. (hin basement membrane disease is not usually associated with renal$ailure.ibliography.<usmano =! <higgeri <4! ;aridi <. Nephronophthisis-medullary cystic disease: clinical andgenetic aspects. 5 Nephrol. 669:##-.

Item 59

Answer: Fright or 'ansel stain o$ the urine sedimentEducational Objective: =ecogni2e the clinical presentation o$ acute interstitial nephritis.(his patient! who was treated with tobramycin and a penicillin congener! developed nonoliguricrenal $ailure associated with rash! $ever! and microscopic hematuria despite resolution o$ her

pulmonary symptoms. (his is a classic picture o$ allergic interstitial nephritis. (he $inding o$urinary eosinophilia on Fright or 'ansel stain o$ the urine sediment will help support thediagnosis. Although aminoglycosides typically cause nonoliguric renal $ailure! they are notassociated with microhematuria! $ever! or rash. 3ntravenous pye-lography is relativelycontraindicated in the presence o$ renal insu$$iciency! and urinary calcium and o>alate levelsare o$ no value in a patient with acute interstitial nephritis.ibliography.Appel <. Acute interstitial nephritis. 3n: Neilson E<! ;ouser F<! eds. 3mmunologic =enalDiseases. "hiladelphia: +ippincott =aven "ress9667:##-%C. U3: 68&76#.=u$$ing 1A! 'oppes "! lend D! ;ugino A! 5arjoura D! Fhittier ;. Eosinophils in urinerevisited. ;lin Neph. 669:8%-8.

Item 60

Answer: ;

locade o$ distal renal tubule potassium e>cretion due to trimethoprimEducational Objective: Understand that patients who receive high-dose trimethoprimJsul$ametho>a2ole can become hyperalemic because o$ reduced renal potassium e>cretioncaused by the trimethoprim."atients with '3/ in$ection can develop various electrolyte abnormalities that are relateddirectly to the viral in$ection! are due to secondary in$ection! and*or are due to medicationsused to treat the '3/ in$ection and the secondary in$ections. (rimethoprim can act similarly topotassium-sparing diuretics when administered in high doses. 3t causes hyperalemia bymechanisms similar to those produced by amiloride or triamterene. (his is the most lielye>planation $or the hyperalemia that developed in this patient. "neumocystis cariniipulmonary in$ection does not usually cause enough parenchymal destruction to generatehyperalemia. (he degree o$ renal insu$$iciency is modest and would not be e>pected toproduce hyperalemia. Although sul$onamides can cause interstitial nephritis and variousassociated electrolyte abnormalities! this is a much less liely diagnosis! and nothing in the

patient,s description suggests an allergy ?no rash! eosinophilia! or eosinophiluria@. Addison,sdisease can be associated with adrenal '3/ in$ection! but this is uncommon and this patient,sblood pressure is normal. (he time course over which hyperalemia developed is much moreconsistent with a trimethoprim e$$ect.ibliography.Alappan =! "era2ella 4A! uller <1. 'yperalemia in hos-pitali2ed patients treated withtrimethoprim-sul$ametho>a2ole. Ann 3ntern 4ed. 6689#:%8-#&.#./ela2)ue2 '! "era2ella 4A! Fright 0! Ellison D'. =enal mechanism o$ trimethoprim-inducedhyperalemia. Ann 3ntern 4ed. 66%96:#68-%&.%.<reenberg 0! =eiser 3F! ;hou 0! "orush 5<. (rimethoprim-sul$ametho>a2ole induces

Page 32: Neurology Answers

8/11/2019 Neurology Answers

http://slidepdf.com/reader/full/neurology-answers 32/43

reversible hyperalemia. Ann 3ntern 4ed. 66%96:#6-C.

Item 61

Answer: Antineutrophil cytoplasmic antibody ?AN;A@ and anti-glomerular basement membrane ?anti-<4@ antibody assaysEducational Objective: Evaluate appropriate serologic tests used to diagnose patients with

pulmonary-renal syndromes.(his patient has a pulmonary-renal syndrome with an in$iltrate on a chest radiograph andevidence o$ renal dys$unction with elevated blood urea nitrogen and serum creatinine levels.(he erythrocyte casts point to a glomerular cause. AntiJglomerular basement membrane?anti-<4@ disease with pulmonary involvement ?<oodpasture,s syndrome@ and antineutrophilcytoplasmic antibody ?AN;A@Jpositive rapidly progressive glomerulonephritis can present thisway. (here$ore! AN;A and anti-<4 assays are appropriate. '3/ nephropathy is the mostcommon renal disease occurring in patients in$ected with '3/. 3t is $ound almost e>clusively inblac patients. +arge echogenic idneys are seen on ultrasonography. Although '3/nephropathy progresses to renal $ailure more )uicly than many other $orms o$ glomerulardisease! it would be unliely to do so in # months. Diseases associated with abnormalelectrophoretic patterns and with hepatitis also do not present with both pulmonary and renal$indings in this way.ibliography

.Appel <. <lomerulonephritis. 3n <oldman +! ennett 5;! eds. ;ecil (e>tboo o$ 4edicine.#st ed. "hiladelphia: F 0aunders9 #&&&:C8-6. 3U: &&%%&8#.5ennette 5;! al =5. 0mall-vessel vasculitis. N Engl 5 4ed. 6679%%7:C#-#%.%.5ennete 5;! Filman A0! al =5. Diagnostic predictive value o$ AN;A serology. 1idney 3nt.669C%:768-.

Item 62

Answer: A;yclophosphamide and glucocorticoidsEducational Objective: 1now the appropriate treatment $or antineutrophil cytoplasmicantibody ?AN;A@Jpositive pauci-immune glomerulonephritis.(he biopsy specimen shows pauci-immune rapidly progressive glomerulonephritis! which istypical o$ AN;A-associated glomerular disease ?either microscopic polyarteritis or Fegener,sgranulomatosis@. (he treatment o$ choice is either oral or intravenous cyclophosphamide plus

glucocorticoids. "lasmapheresis! which has been shown to be e$$ective in patients with<oodpasture,s syndrome! has not been proved e$$ective in patients with AN;A-associatedglomerular disease. ;yclosporine has no proven role in the acute treatment o$ these diseasesat this time.ibliography.5ennette 5;! al =. 0mall-vessel vasculitis. N Engl 5 4ed. 6679%%7:C#-#%.#.Nachman "'! 'ogan 0+! 5ennette 5;! al =5. (reatment response and relapse inantineutrophil cytoplasmic autoantibody-associated microscopic polyangiitis andglomerulonephritis. 5 Am 0oc Neph. 66897:%%-6.

Item 63

Answer: A(he presence o$ microalbuminuria in a patient with type or type # diabetes mellitus isusually indicative o$ early diabetic nephropathy

Educational Objective: Understand how to measure microalbuminuria and now the clinicalimplications o$ microalbuminuria regarding therapy and ris o$ renal and cardiovasculardisease.3ncreased urinary protein e>cretion is the earliest marer o$ diabetic nephropathy. 'owever!the urine dipstic measurement is a relatively insensitive marer because it does not becomepositive until protein e>cretion e>ceeds %&& mg*d. A speci$ic assay $or albumin is moresensitive because the normal rate o$ albumin e>cretion is less than #& mg*d. "ersistent valuesbetween %& and %&& mg*d in a patient with diabetes mellitus is termed RmicroalbuminuriaSand is usually indicative o$ early diabetic nephropathy. "atients with values above %&& mg*dare considered to have overt macroscopic proteinuria.

Page 33: Neurology Answers

8/11/2019 Neurology Answers

http://slidepdf.com/reader/full/neurology-answers 33/43

A #-hour urine collection is the gold standard $or the detection o$ microalbuminuria. 'owever!screening can be done simply by using a timed urine collection or an early morning specimento minimi2e changes in urine volume. Use o$ the albumin:creatinine ratio has recently beenrecommended as the pre$erred screening strategy $or all patients with diabetes mellitus. Analbumin:creatinine ratio above %& mg*g represents an albumin e>cretion o$ more than %&mg*d and is diagnostic o$ microalbuminuria.Not all patients with diabetes mellitus and microalbuminuria develop overt nephropathy over

the ne>t decade. +ess than C&G o$ patients with type diabetes mellitus andmicroalbuminuria are at ris $or progression. "rogression to overt nephropathy occurs in #&Gto &G o$ white patients with type # diabetes mellitus. =is $actors contributing to progressioninclude hyperglycemia! hypertension! and tobacco use.(he presence o$ microalbuminuria is an important ris $actor $or the development o$cardiovascular disease and early cardiovascular mortality in patients with both type # diabetesmellitus and essential hypertension. or e>ample! patients with type # diabetes mellitus andmicroalbuminuria who were $ollowed $or %. years had a mortality rate o$ #G compared witha mortality rate o$ G $or patients with type # diabetes mellitus without microalbuminuria. (heincreased ris appeared to be independent o$ other cardiovascular ris $actors.(herapy with the dihydropyridine calcium channel blocer ni$edipine has been shown toincrease proteinuria in patients with type # diabetes mellitus. Optimal therapy $or hypertensivepatients with type # diabetes mellitus and microalbuminuria is an angiotensin-convertingen2yme ?A;E@ inhibitor. "atients treated with an A;E inhibitor had a slower progression to

overt proteinuria and a slower increase in serum creatinine levels over a %-year period.ibliography.1eane F! Enoyan <. "roteinuria! albuminuria! ris! assessment! detection! elimination?"A=ADE@: a position paper o$ the National 1idney oundation. Am 5 1idney Dis.6669%%:&&-&.#.4ogensen ;E! /estbo E! "oulsen "+! ;hristiansen ;! Damsgaard E4! Eisjaer '! et al.4icroalbuminuria and potential con$ounders. A review and some observations on variability o$urinary albumin e>cretion. Diabetes ;are. 66C9:C7#-.%.4ogensen ;E. 4icroalbuminuria predicts clinical proteinuria and early mortality in maturity-onset diabetes. N Engl 5 4ed. 69%&:%C8-8&..1lein =! 1lein E! 4oss 0E! ;ruicshans 15. (en-year incidence o$ gross proteinuria inpeople with diabetes. Diabetes. 66C9:68-#%.C.4attoc 4! 4orrish N5! /iberti <! 1een! '! it2gerald A"! 5acson <. "rospective study o$microalbuminuria as predictor o$ mortality in N3DD4. Diabetes. 66#9:7%8-.

8.+ebovit2 'E! Fiegmann (! ;naan A! 0hahin$ar 0! 0ica DA! roadstone /! et al. =enalprotective e$$ects o$ enalapril in hypertensive N3DD4: role o$ baseline albuminuria. 1idney 3nt0uppl. 669C:0C&-C.

Item 64

Answer: Advise him that this is a benign $inding and that although it may be a ris $actor $ornephrolithiasis! it never leads to progressive renal $ailureEducational Objective: Understand the clinical $eatures and course o$ medullary spongeidney.4edullary sponge idney is a relatively benign condition that at times is associated withmicroscopic hematuria and occasionally with stone $ormation. 'owever! it never leads toprogressive renal $ailure and re)uires no treatment. 3n contrast! a disease with a similarsounding name! medullary cystic idney disease! is a genetic cystic disorder that presents in

childhood and typically leads to renal $ailure. (he role o$ angiotensin-converting en2ymeinhibitors is unproved in patients with renal disorders other than hypertension or glomerulardiseases.ibliography.<usmano =! <higgeri <4! ;aridi <. Nephronophthisis-medullary cystic disease: clinical andgenetic aspects. 5 Nephrol. 669:##-.

Item 65

Answer: ;

Page 34: Neurology Answers

8/11/2019 Neurology Answers

http://slidepdf.com/reader/full/neurology-answers 34/43

=enal tubular acidosisEducational Objective: Di$$erentiate among the causes o$ nephrolithiasis and recogni2e theclinical $eatures o$ renal tubular acidosis.=enal tubular acidosis occurs more commonly in women than in men. (ypically! the serumbicarbonate concentration is low and the serum chloride concentration is elevated! re$lecting anon-anion gap metabolic acidosis. (he urinary p' is inappropriately alaline ?L C.C@ in view o$the metabolic acidosis! and hypoalemia is $re)uently present. =enal tubular acidosis is

associated with several systemic disorders! including in$lammatory bowel disease and 0j 

gren,ssyndrome. Nephro-calcinosis ?multiple small calyceal and medullary stones seen onradiographs o$ the idneys@ is common in patients with renal tubular acidosis. ;ystinuria is notassociated with systemic acidosis! hypoalemia! or nephrocalcinosis. 0truvite stones arealways associated with in$ected alaline urine. 4edullary sponge idney may be associatedwith nephro-calcinosis but does not usually cause metabolic acidosis or hypoalemia.ibliography.ucalew /4 5r. Nephrolithiasis in renal tubular acidosis. 5 Urol. 669:7%-7.#.0mulders 4! rissen "'! 0laats E'! 0ilberbusch 5. =enal tubular acidosis. "athophysiologyand diagnosis. Arch 3ntern 4ed. 6689C8:8#6-%8.

Item 66

Answer: Urine immunoelectrophoresis

Educational Objective: =ecogni2e the clinical presentation o$ renal disease in patients withmyeloma.(his older man has anemia out o$ proportion to his renal insu$$iciency! a low anion gap!hypercalcemia in the presence o$ an elevated serum phosphorus level! and #. g*# h o$urinary protein! although the dipstic test is only trace positive $or protein. All point tomyeloma cast nephropathy as the cause o$ the renal insu$$iciency. (here$ore! urineimmunoelectrophoresis should be done ne>t. Fhen analy2ed by immunoelectrophoresis! theelevated urinary protein will be $ound to be due to abnormal light chains. 3ntravenouspyelography is relatively contraindicated in patients with reduced renal $unction! as this studymay result in acute renal $ailure. 0erum angiotensin-converting en2yme levels are increased inpatients with sarcoid! which can be associated with hypercalcemia and renal disease! but notwith the other $eatures noted in this patient. 0erum complement determination andantinuclear antibody assay are most use$ul in diagnosing glomerular diseases in patients withalbuminuria.

ibliography.1yle =A. 0e)uence o$ testing $or monoclonal gammopathies. Arch "athol +ab 4ed.6669#%:-.

Item 67

Answer: AObservation and repeat arterial blood gas studies in # hoursEducational Objective: Understand that transient lactic acidosis $re)uently occurs a$ter asei2ure and does not re)uire any speci$ic therapy! and recogni2e that an osmotic gap e>istswhenever signi$icant blood levels o$ ethanol are present."ostictal patients typically develop a transient lactic ?anion gap@ metabolic acidosis thatresolves without treatment over 8& to 6& minutes. (his acidosis is caused by acceleratedproduction o$ lactic acid in muscle and reduced hepatic lactate uptae. (he acidosis does notre)uire speci$ic therapy and resolves a$ter sei2ure control. (his patient also has an osmotic

gap that is due to his blood ethanol level. 3$ his blood ethanol value had been normal!ingestion o$ a to>ic poison such as methanol or ethylene glycol would need to be considered.Under such circumstances! $omepi2ole or ethanol may be administered. (hese drugs inhibitalcohol dehydrogenase and thereby slow the metabolism o$ certain to>ic compounds to moreto>ic metabolites. 'owever! the osmotic gap is easily e>plained in this patient.

ibliography.Orringer ;E! Eustace 5;! Funsch ;D! <ardner +. Natural history o$ lactic acidosis a$tergrand-mal sei2ures. A model $or the study o$ an anion-gap acidosis not associated with

Page 35: Neurology Answers

8/11/2019 Neurology Answers

http://slidepdf.com/reader/full/neurology-answers 35/43

hyperalemia. N Engl 5 4ed. 6779#67:768-6.#.Osterloh 5D! 1elly (5! 1hayam-ashi '! =omeo =. Discrepancies in osmolal gaps andcalculated alcohol concentrations. Arch "athol +ab 4ed. 6689#&:8%7-.

Item 68

Answer: (herapy with an angiotensin-converting en2yme ?A;E@ inhibitor is liely to reduce the degree

o$ microalbuminuria and signi$icantly lessen the chance o$ progression to overt nephropathy inpatients with type diabetes mellitus and microalbuminuriaEducational Objective: =ecogni2e microalbuminuria as a ris $actor $or developing type diabetic nephropathy and understand the importance o$ angiotensin-converting en2ymeinhibitor therapy in reducing this ris.4icroalbuminuria is the earliest clinical sign o$ type diabetic nephropathy. +ess than C&G o$patients with type diabetes mellitus and microalbuminuria are at ris $or developing overtnephropathy. 3ncreasing attention to control o$ both hypertension Wparticularly withangiotensin-converting en2yme ?A;E@ inhibitorsX and hyperglycemia appears to contribute tothe improvement in the course o$ this disease. (herapy with A;E inhibitors $or patients withtype diabetes mellitus! microalbuminuria! and normal blood pressure will reduce the degreeo$ microalbuminuria and decrease the chance that the patient will develop overt nephropathy.(hree recent trials involving more than && patients with type diabetes mellitus! normalblood pressure! and micro-albuminuria have demonstrated that the degree o$ microalbu-

minuria is signi$icantly reduced by appro>imately &G in patients receiving an A;E inhibitorcompared with a CG increase per year in patients receiving a placebo. "rogression to overtproteinuria was maredly reduced at # years in the A;E inhibitor group compared with theplacebo group ?7G versus #%G@. 'ence! A;E inhibitors are recommended $or all patients withtype diabetes mellitus and microalbuminuria whether or not their blood pressure is elevated.As mentioned above! less than C&G o$ patients with type diabetes mellitus andmicroalbuminuria develop overt nephropathy. 3n $act! patients who develop microalbuminuriaa$ter having had diabetes $or C or more years actually have less ris o$ developing overtnephropathy ?only an G ris according to one study@.(he development o$ microalbuminuria does correlate with glycemic control. A recentretrospective study e>amined both albumin e>cretion and glycemic control in more than 8&&patients with type diabetes mellitus. (he ris o$ having microalbuminuria increased abruptlywhen hemoglobin A; values were above G. Although this retrospective study does notprove cause and e$$ect! the data are compatible with the importance o$ glycemic control. (he

importance o$ glycemic control in the secondary prevention o$ diabetic nephropathy was bestdemonstrated in the Diabetes ;ontrol and ;omplications (rial ?D;;(@. Among patients withmicroalbuminuria at entry! albumin e>cretion increased slightly over time in patients who werereceiving conventional insulin therapy but did not increase at all in patients who were receivingintensive insulin therapy. (his bene$it was associated with a lower rate o$ progression to overtnephropathy.Not all patients with type diabetes mellitus appear to be candidates $or A;E inhibitor therapyi$ both blood pressure and albumin e>cretion are normal. A subset o$ patients with type diabetes mellitus who had normal blood pressure and normal albumin e>cretion ?baseline mg*d@ were randomly assigned to receive lisinopril or placebo. (here was a slight increase inalbumin e>cretion over time in both groups with an insigni$icant di$$erence between thelisinopril and placebo groups. 'ence! scienti$ic data are lacing to support the hypothesis thatpatients with type diabetes mellitus who have both normal blood pressure and albumine>cretion will bene$it $rom A;E inhibitor therapy.

ibliography.1rolewsi A0! Farram 5'! =and +3! 1ahn ;=. Epidemiologic approach to the etiology o$ type3 diabetes mellitus and its complications. N Engl 5 4ed. 679%7:%6&-.#.4ogensen ;E. 4icroalbuminuria as a predictor o$ clinical diabetic nephropathy. 1idney 3nt.679%:87%-6.%.1rolewsi A0! +a$$el +4! 1rolewsi 4! uinn 4! Farram! 5'. <lycosylated hemoglobin andthe ris o$ microalbuminuria in patients with insulin-dependent diabetes mellitus. N Engl 54ed. 66C9%%#:#C-C..E$$ect o$ intensive therapy on the development and progression o$ diabetic nephropathy inthe Diabetes ;ontrol and ;omplications (rial. (he Diabetes ;ontrol and ;omplications ?D;;(@

Page 36: Neurology Answers

8/11/2019 Neurology Answers

http://slidepdf.com/reader/full/neurology-answers 36/43

=esearch <roup. 1idney 3nt. 66C97:7&%-#&.C.;aptopril reduces the ris o$ nephropathy in 3DD4 patients with microalbuminuria. (he4icroalbuminuria ;aptopril 0tudy <roup. Diabetologia. 6689%6:C7-6%.8./iberti <! 4ogensen ;E! <roop +;! "auls! 5. E$$ect o$ captopril on progression to clinicalproteinuria in patients with insulin-dependent diabetes mellitus and microalbuminuria.European 4icroalbuminuria ;aptopril 0tudy <roup. 5A4A. 669 #7:#7C-6.7.=andomised placebo-controlled trial o$ lisinopril in normotensive patients with insulin-

dependent diabetes and normoalbuminuria or microalbuminuria. (he EU;+3D 0tudy <roup.+ancet. 6679%6:77-6#.

Item 69

Answer: A3ncreased tubular secretion o$ creatinineEducational Objective: Understand that creatinine clearance overestimates the trueglomerular $iltration rate in patients with renal $ailure.3n assessing idney $unction! comparisons o$ creatinine clearance with simultaneousmeasurements o$ the glomerular $iltration rate ?<=@ have shown that the creatinine clearancevalue e>ceeds the <=. (his overestimation may be as great as &G to #&G in individualswith normal renal $unction and &G to &&G in patients with a2otemia. (he main reason isthat tubular secretion o$ creatinine occurs in all patients but is higher and more variable asrenal $unction declines. =enal tubular reabsorption o$ creatinine does not occur. 3ncreased

release o$ creatinine $rom muscle occurs in muscle injury ?rhabdomyolysis@ but is not liely inthis patient.ibliography.Falser 4! Drew ''! +arance ND. ;reatinine measurements o$ten yield $alse estimates o$progression in chronic renal $ailure. 1idney 3nt. 69%:#-.

Item 70

Answer: D3$os$amide interstitial nephritisEducational Objective: =ecogni2e the clinical picture o$ chemotherapeutic agent-induced renal$ailure.(his patient has i$os$amide nephropathy. 3$os$amide can cause chronic interstitial damage andprogressive renal disease with a bland urinary sediment. (his patient,s urinary sediment doesnot suggest a glomerular disease ?$ew erythrocytes and no erythrocyte casts@! maing

poststreptococcal disease and 3gA nephropathy unliely. =adiation nephritis would not occurunless the $ield o$ radiation includes the idneys. 3t would also cause progressive renalinsu$$iciency and hypertension.ibliography.<oren 4"! Fright =1! "ratt ;! 'orowit2 4E! Dodge =1! et al. "otentiation o$ i$os$amideneuroto>icity! hematoto>icity! and tubular nephroto>icity by prior cisdiamminedichloroplatinum?33@ therapy. ;ancer =es. 6797:C7-8&.#.'o "(! Bimmerman 1! Fe>ler +'! laney 0! 5arosinsi "! Feaver-4c;lure +! et al. Aprospective evaluation o$ i$os$amide-related nephroto>icity in children and young adults.;ancer. 66C978:#CC7-8.%.Fright 5E! Elias A! (retyaov O! 'olden 0! Andersen 5! Fheeler ;! et al. 'igh-dosei$os$amide! carboplatin! and etoposide pharmacoinetics: correlation o$ plasma drug levelswith renal to>icity. ;ancer ;hemother "harmacol. 66C9%8:%C-C.

Item 71

Answer: ;=enal calculus

Page 37: Neurology Answers

8/11/2019 Neurology Answers

http://slidepdf.com/reader/full/neurology-answers 37/43

Educational Objective: =ecogni2e the reason $or the abrupt loss o$ renal $unction occurring ina patient with slowly progressive autosomal dominant polycystic idney disease.Autosomal dominant polycystic idney disease ?AD"1D@ may present in young adults withhematuria! hypertension! a2otemia! or palpable idneys. (he loss o$ renal $unction due toprogressive cystic replacement o$ the renal parenchyma is slow ?o$ten measured in years oreven decades@. (he clinical course may be complicated by urinary tract in$ections! cystin$ections! or hematuria due to cyst hemorrhage or renal calculi.

(his patient has had a precipitous reduction in renal $unction! as his serum creatinine level hasmore than doubled in month and is accompanied by microhematuria and le$t $lan pain. Aplain radiograph o$ his abdomen and pelvis did not demonstrate any radiopa)ue calculi! but ;(showed moderate obstruction o$ the le$t renal collecting system. (he patient had unilateralsubtotal obstruction on the le$t due to a C-mm uric acid calculus. 1idney stones occur in up to#&G o$ patients with AD"1D. 3n contrast to the predominance o$ calcium o>alate stones inidiopathic stone $ormers! more than C&G o$ the stones in patients with AD"1D are composedo$ uric acidJo$ten in the absence o$ clinical gout or hyperuricemia. (he uric acid calculi areradiolucent on radiographs! but have a characteristic density on renal ultrasonography or ;(that usually distinguishes them $rom tumor! blood clot! $ungus ball! or other potentialobstructing lesions. (he uric acid calculi may be passed spontaneously or dissolve withalalini2ation o$ the urine.(he loss o$ renal $unction is much too rapid to be accounted $or by natural progression o$AD"1D. (he patient,s renal $unction has been relatively stable $or # years while on an

angiotensin-converting en2yme ?A;E@ inhibitor $or his hypertension so that his medication isprobably not causing the precipitous decline. 'owever! abrupt loss o$ renal $unction has beenreported in a small series o$ patients with very severe polycystic idney disease who weretaing A;E inhibitors. inally! neither the urinalysis nor the ;( is suggestive o$ urinary tractin$ection or cyst hemorrhage.ibliography."errone =D. E>trarenal mani$estations o$ AD"1D. 1idney 3nt. 6679C:#&##-%8.#.<abow "A. Autosomal dominant polycystic idney disease. N Engl 5 4ed. 66%9%#6:%%#-#.%.<abow "A. Autosomal dominant polycystic idney diseaseJmore than a renal disease. Am 51idney Dis. 66&98:&%-%..(orres /E! Filson D4! 'attery ==! 0egura 5F. =enal stone disease in autosomal dominantpolycystic idney disease. Am 5 1idney Dis. 66%9##:C%-6.

Item 72

Answer: ;;omplication o$ erythropoietin administrationEducational Objective: =ecogni2e hypertension as a complication o$ erythropoietin therapy.(his patient most liely has erythropoietin-induced hypertension. 'is hypertension developeda$ter a short period o$ moderate-dose subcutaneous erythropoietin administration and a$ter arapid rise in hematocrit. Appro>imately #&G to C&G o$ patients who receive erythropoietinintravenously $or the anemia o$ chronic renal $ailure develop a diastolic blood pressureelevation o$ & mm 'g or more. (his occurs somewhat less o$ten in patients who are givensubcutaneous erythropoietin. actors contributing to the hypertensive response include directvasoconstrictive e$$ect! increase in whole blood viscosity! blood volume e>pansion! priorpersonal or $amily history o$ hypertension! diminished response to nitric o>ide and increasedresponse to norepinephrine! increased plasma endothelin levels! and mared elevation o$intracellular calcium levels. Erythropoietin-induced hypertension can be prevented byincreasing the hematocrit slowly by %&G to %CG. "rior to dialysis! hypertensive patients can

be treated with diuretics and antihypertensive agents. 3n severe cases! erythropoietin mayneed to be decreased or discontinued.(his patient is unliely to have worsening renal $unction because his serum creatinineconcentration remained stable and there is only slight edema without signs o$ overt volumeoverload. Essential hypertension and ;ushing,s syndrome are also unliely because o$ the time$rame during which the hypertension occurred! which was a$ter erythropoietin administrationwas started.ibliography.Nissenson A=. Achieving target hematocrit in dialysis patients: new concepts in ironmanagement. Am 5 1idney Dis. 6679%&:6&7-.

Page 38: Neurology Answers

8/11/2019 Neurology Answers

http://slidepdf.com/reader/full/neurology-answers 38/43

#.0under-"lassmann <! 'orl F'. +aboratory diagnosis o$ anaemia in dialysis patients: use o$common laboratory tests. ;urr Opin Nephrol 'ypertens. 66798:C88-6.%.4ittman N! 0reedhara =! 4ushnic =! ;hattopadhyay 5! Belmanovic D! /aseghi 4! et al.=eticulocyte hemoglobin content predicts $unctional iron de$iciency in hemodialysis patientsreceiving r'uE"O. Am 5 1idney Dis. 6679%&:6#-##..ovy ;! (sobo ;! ;rapan2ano +! =orive <! eguin ! Albert A! et al. actors determining thepercentage o$ hypochromic red blood cells in hemodialysis patients. 1idney 3nt.

6669C8:%-6.C.N1-DO3 clinical practice guidelines $or the treatment o$ anemia o$ chronic renal $ailure.National 1idney oundation-Dialysis Outcomes uality 3nitiative. Am 5 1idney Dis. 667Oct9%&? 0uppl %@:06#-#&.

Item 73

Answer: ;;( o$ the idneys and cystoscopyEducational Objective: Evaluate the cause o$ gross hematuria in a patient with sicle celldisease."ainless gross hematuria due to medullary in$arcts is common in patients with both sicle celldisease and sicle cell trait. All adult patients with these disorders should be evaluated by ;(o$ the idneys and cystoscopy to rule out other pathologic processes be$ore this diagnosis o$e>clusion is made. ;( will rule out upper urinary tract lesions and liely show papillary

necrosis! and cystoscopy will e>clude lower urinary tract lesions. Although ultrasonographymay show stones and at time show papillary necrosis! this study is not as sensitive as ;(.Angiography will de$ine the renal vasculature but is an invasive procedure. Neitherultrasonography nor angiography can rule out a bladder or urinary tract lesion.ibliography."ham "(! "ham ";! Filinson A'! +ew 0. =enal abnormalities in sicle cell disease. 1idney3nt. #&&&9C7:-.#.0cheinman 53. 0icle cell nephropathy. 3n: <reenberg A! ed. 1idney Diseases. #nd ed. 0anDiego: Academic "ress9 66:%&6-%. U3: 67788

Item 74

Answer: 0peci$ic <ravity.&&"rotein #I

lood Negative4icroscopic E>amination -% leuocytes! C-& renal tubular cells*hp$9 many pigmentedgranular casts! occasional renal tubular cell casts9 'anselYs stain negativeEducational Objective: =ecogni2e the characteristics o$ the urine in patients with acutetubular necrosis and other causes o$ acute renal $ailure.3n contrast-induced acute tubular necrosis ?A(N@ the urine may be almost normal but o$tenshows muddy brown casts! renal tubular cells! and renal tubular cell casts. (here usually is nosigni$icant proteinuria. 3n this patient! the proteinuria re$lects the underlying renal disease! butthe sediment is typical o$ A(N. Erythrocyte casts are not seen in patients with A(N. +euocytesaccompanied by bacteria are typical o$ urinary tract in$ection but not o$ A(N. Urinaryeosinophils may be $ound in patients with acute interstitial nephritis! atheroembolic renaldisease! and urinary tract in$ections. 3n a study by ;orwin et al ?Arch 3ntern 4ed.6C9C:&67@! 8G o$ patients whose urine was speci$ically e>amined had urinaryeosinophils. O$ these! CG had clinical evidence o$ upper or lower urinary tract in$ection! and

G had clinical or biopsy evidence o$ acute interstitial nephritis. 3n several small studies!eosinophiluria has been observed in patients with atheroembolic renal disease. (hus! the$inding o$ eosinophiluria in patients with renal disease is nonspeci$ic and must be interpretedbased on the clinical conte>t o$ the particular patient.ibliography.4iller (=! Anderson =5! +inas 0+! 'enrich F+! erns A0! <abow! "A! 0chrier =F. Urinarydiagnostic indices in acute renal $ailure: a prospective study. Ann 3ntern 4ed. 6796:7-C&.Item 75

Answer: DAs the patient again about use o$ prescribed drugs and over-the-counter medications

Page 39: Neurology Answers

8/11/2019 Neurology Answers

http://slidepdf.com/reader/full/neurology-answers 39/43

Educational Objective: =ecogni2e the clinical picture o$ analgesic nephropathy.3n patients with chronic bac pain! arthritis! and headaches! use o$ over-the-counteranalgesics and nonsteroidal anti-in$lammatory drugs is very common. (here$ore! the patientshould be ased again about use o$ these medications! as chronic use can lead to interstitialdamage and renal dys$unction. 3ntravenous pyelography will show papillary necrosis9 renalultrasonography may also show this $inding. Other diseases associated with papillary necrosisinclude sicle cell disease and diabetes mellitus. 'owever! neither is a possibility in this

patient! since she is white! is not anemic! and is not diabetic. =enal tuberculosis can mimicpapillary necrosis on intravenous pyelography but rarely causes renal dys$unction in patients inFestern countries.ibliography.'enrich F+! Agodoa +E! arrett ! ennett F4! lant2 =;! ucalew /4 5r! et al. Analgesicsand the idney. 0ummary and recommendations to the 0cienti$ic Advisory oard o$ theNational 1idney oundation $rom an Ad 'oc ;ommittee o$ the National 1idney oundation. Am5 1idney Dis. 6689#7:8#-C.#.De roe 4! Elseviers 44. Analgesic nephropathy. N Engl 5 4ed. 669%%:8-C#.

Item 76

Answer: A(he patient is liely magnesium depleted! which prevents potassium repletionEducational Objective: 1now that chronic diarrhea disorders may result in multiple ion

de$iciencies and that coe>isting hypoalemia! hypocalcemia! and hypophosphatemia shouldalways raise the strong possibility o$ simultaneous magnesium depletion. Also understand thatcorrection o$ hypoalemia and hypocalcemia is di$$icult unless magnesium depletion is alsocorrected.(he patient has magnesium depletion. <astrointestinal disorders are a common cause o$magnesium depletion! which is o$ten associated with potassium de$icits ?due to increased renalpotassium loss@ and hypocalcemia ?due to $unctional hypoparathyroidism@. (he hypoalemiaand hypocalcemia are very di$$icult to correct unless the magnesium de$icit is $irst addressed.4agnesium! potassium! and phosphorus depletion may occur simultaneously in patients withchronic diarrhea.ibliography.0hils 4E. E>perimental human magnesium depletion. 4edicine. 6869:8-C.#.Fhang =! Fhang DD! =yan 4". =e$ractory potassium repletion. A conse)uence o$magnesium de$iciency. Arch 3ntern 4ed. 66#9C#:&-C.

%.Fhang =. 4agnesium de$iciency: pathogenesis! prevalence! and clinical implications. Am 54ed. 679#:#-6.

Item 77

Answer: A;alcium-containing phosphate binders with mealsEducational Objective: 1now the appropriate therapy $or controlling hyperphosphatemia in apatient with chronic renal $ailure.(his patient has hyperphosphatemia and a normal serum calcium level ?adjusted $or a lowserum albumin level@ along with renal insu$$iciency and an elevated parathyroid hormone levelconsistent with secondary hyperparathyroidism. (he optimal treatment is dietary phosphorusrestriction and oral phosphate binders. Agents such as calcium acetate ?"hos+o@ or calciumcarbonate ?O0-;al@ act as phosphate binders when taen with meals. (he ris o$ developinghypercalcemia because o$ this therapy is minimal since only a small amount o$ calcium is

absorbed $rom the gastrointestinal tract when the calcium-containing binders are taen withmeals. 3n $act! calcium carbonate should be administered on an empty stomach when beinggiven to treat hypocalcemia. 3$ hypercalcemia does develop! a new nonabsorbable polymerthat binds phosphate in the gastrointestinal tract is now available ?sevelamer! =ena<el@. 3t isas e$$ective as calcium-containing binders without having an adverse e$$ect on serum calci- umlevels and also appears to lower plasma cholesterol levels. 0evelamer re)uires multiple-dosetablets with meals and may be associated with some gastrointestinal side e$$ects.(his patient should not receive !#C-dihydro>yvitamin D because this will promote calciumreabsorption $rom the gastrointestinal tract ?and possibly promote hypercalcemia@ and doesnot bind dietary phosphorus in the gastrointestinal tract. Although aluminum hydro>ide may

Page 40: Neurology Answers

8/11/2019 Neurology Answers

http://slidepdf.com/reader/full/neurology-answers 40/43

be used as a phosphate binder! its use has been discontinued because o$ potential side e$$ectsdue to aluminum into>ication. 4agnesium o>ide is not an acceptable phosphate binder andmay produce dangerous symptomatic hypermagnesemia in a patient with chronic renal $ailure.ibliography.0latopolsy EA! ure 01! Dillon 4A. =ena<el! a nonabsorbed calcium- and aluminum-$reephosphate binder! lowers serum phosphorus and parathyroid hormone. (he =ena<el 0tudy<roup. 1idney 3nt. 6669CC:#66-%&7.

#.;hertow <4! Dillon 4! ure 01! 0teg 4! leyer A5! <arrett N! et al. A randomi2ed trial o$sevelamer hydrochloride ?=ena<el@ with and without supplemental calcium. 0trategies $or thecontrol o$ hyperphosphatemia and hyperparathyroidism in hemodialysis patients. ;lin Nephrol.6669C:-#8.

Item 78

Answer: AEthylene glycolEducational Objective: =ecogni2e the typical presenting $eatures o$ ethylene glycol poisoning.(his patient,s history o$ having drun some RpoisonS and the $indings o$ a mared anion gapmetabolic acidosis! slightly increased blood urea nitrogen and serum creatinine levels! andmany rectangular crystals in his urine are strongly suggestive o$ ethylene glycol poisoning.(his diagnosis must be established promptly so that appropriate therapy can be initiated.Ethylene glycol itsel$ is only moderately to>ic! but its metabolic products! which include o>alic!

glyo>ylic! and glycolic acid! are e>tremely to>ic and cause metabolic acidosis and tissuedamage. (here$ore! the treatment regimen always includes inhibition o$ alcoholdehydrogenase! the initial en2yme responsible $or the metabolism o$ ethylene glycol. (his canbe accomplished with ethanol! which is a competitive inhibitor o$ the en2yme! or with$omepi2ole! which is a more potent alcohol dehydrogenase inhibitor. (reatment o$ten alsore)uires the prompt initiation o$ hemodialysis. Although methanol poisoning also causes ananion gap metabolic acidosis! urine crystals are not generally seen and renal insu$$iciency isless common. 3sopropyl alcohol ingestion may cause somnolence and coma and the smell o$acetone on the breath but does not generally lead to metabolic acidosis ?unless the patientdevelops cardiovascular collapse@ or urine crystal $ormation. ;yanide ingestion causes severelactic -acidosis! but urine crystals are not seen. ;alcium o>alate is insoluble! not wellabsorbed! and would not cause metabolic acidosis.ibliography.Emmett 4! Narins =<. ;linical use o$ the anion gap. 4edicine. 6779C8:%-C.

#.rent 5! 4c4artin 1! "hillips 0! urhart 11! Donovan 5F! Fells 4! et al. omepi2ole $or thetreatment o$ ethylene glycol poisoning. 4ethylpyra2ole $or (o>ic Alcohols 0tudy <roup. N Engl5 4ed. 6669%&:%#-.%.<abow "A! ;lay 1! 0ullivan 5! +epo$$ =. Organic acids in -ethylene glycol into>ication. Ann3ntern 4ed. 689&C:8-#&.

Item 79

Answer: ;(his patientYs antihypertensive medications may increase norepinephrine levels! and the testshould be repeated a$ter proper adjustments o$ medicationsEducational Objective: =ecogni2e that common medications may inter$ere with the diagnosiso$ pheochromocytoma."heochromocytoma is an unusual cause o$ hypertension that o$ten is characteri2ed by episodicsymptoms o$ headache! nausea! tremor! diaphoresis! and tachycardia. (he diagnosis is

supported by the $indings o$ elevated plasma or urinary catecholamine levels in the absence o$ secondary causes o$ such elevations. 3n this patient! the diagnosis o$ pheochromo-cytoma ismistaenly suggested by the spurious elevation o$ plasma and urinary catecholamine levelsinduced by her antihypertensive medication! labetalol. (here$ore! plasma and urinarycatecholamine measurements should be repeated a$ter her labetalol and diuretic have beenstopped $or # wees. 3$ medication is needed $or hypertension control! a dihydropyridinecalcium channel blocer such as amlodipine would be suitable. Fhen medications werestopped! both plasma and urinary catecholamine levels were normal in this patient who hasessential hypertension and an an>iety disorder. <enerally! drugs such as clonidine and b-blocers suppress catecholamines! whereas diuretics and direct-acting vasodilators may

Page 41: Neurology Answers

8/11/2019 Neurology Answers

http://slidepdf.com/reader/full/neurology-answers 41/43

stimulate catecholamines. +abetalol directly inter$eres with the assay measurements o$ plasmaand urinary catecholamine levels.An elevated plasma glucose level! as well as laboratory evidence o$ hemoconcentration! maybe clues to the diagnosis o$ pheochromocytoma! whether or not this lesion is part o$ a multipleendocrine neoplasia ?4EN@ syndrome. 0elective adrenal vein sampling is re)uired only inpatients with clinical and biochemical evidence o$ pheochromocytoma but with normalradiographic studies. inally! pheochromocytomas! although rare! can occur in any age group.

(he diagnosis is commonly made in the th or Cth decade. (he tumor may -present withorthostatic hypotension. (he reason $or this is not entirely clear! but it may re$lect a relativedepletion in plasma volume.ibliography.0tein ""! lac '=. A simpli$ied diagnostic approach to pheochromocytoma. A review o$ theliterature and report o$ one institution,s e>perience. 4edicine ?altimore@. 6697&:8-88.#.eldman 54. alsely elevated urinary e>cretion o$ catecholamines and metanephrines inpatients receiving labetalol therapy. 5 ;lin "harmacol. 679#7:#-6#.%.Ferbel 00! Ober 1". "heochromocytoma. Update on diagnosis! locali2ation! andmanagement. 4ed ;lin North Am. 66C976:%-C%..ravo E+. "heochromocytoma: new concepts and $uture trends. 1idney 3nt. 669&:C-C8.

Item 80

Answer: ?@/omitingEducational Objective: Evaluate and treat a pregnant woman with acute renal $ailure."regnancy is in$re)uently complicated by acute renal $ailure. /olume depletion secondary tohyperemesis gravidarum is the most liely cause o$ renal insu$$iciency in this patient with signso$ orthostasis and a bland urinalysis. Other causes o$ acute renal $ailure during pregnancyinclude e>acerbation o$ underlying renal disease! such as lupus nephritis and other glomerulardiseases! and acute tubular necrosis related to sepsis. "reeclampsia is unliely because o$ theearly stage o$ pregnancy and the absence o$ hypertension! edema! and proteinuria.(hrombotic microangiopathy usually develops postpartum or in the peripartum period! butoccasionally occurs earlier in pregnancy. 'owever! thrombotic microangiopathy is unliely inthis patient because o$ the bland urinalysis and the absence o$ signs o$ hemolysis on theperipheral blood smear. Urinary tract obstruction is possible but is less liely than prerenala2otemia. (here is no evidence o$ systemic lupus erythematosus! but this disease can occur

during pregnancy.(he patient should be hospitali2ed $or volume repletion! which is both a therapeutic and adiagnostic maneuver. 3$ the clinical and laboratory $eatures o$ volume depletion and renalinsu$$iciency reverse! there will be no need to consider a diagnosis o$ urinary tract obstruction."lasmapheresis is indicated $or the treatment o$ thrombotic microangiopathy duringpregnancy! but there is no sign o$ this disorder in this patient. (here is no indication $ortherapeutic abortion or administration o$ -glucocorticoids.ibliography.N2erue ;4! 'ewan-+owe 1! Nwawa ;. Acute renal $ailure in pregnancy: a review o$ clinicaloutcomes at an inner-city hospital $rom 68-668. 5 Natl 4ed Assoc. 6696&:8-6&.#.1on 0"! 1wan 5(! =a$tery 45. =eversible renal $ailure due to the antiphospholipid antibodysyndrome! pre-eclampsia and renal thrombotic microangiopathy. ;lin Nephrol. 66C9:#7-%.%.5ena 4! 4itch FE. =apidly reversible acute renal $ailure $rom ureteral obstruction inpregnancy. Am 5 1idney Dis. 6689 #:C7-8&.

Item 81

Answer: ?A@/olume repletion with normal saline and provision o$ nutritionEducational Objective: Evaluate and treat a pregnant woman with acute renal $ailure.

Page 42: Neurology Answers

8/11/2019 Neurology Answers

http://slidepdf.com/reader/full/neurology-answers 42/43

"regnancy is in$re)uently complicated by acute renal $ailure. /olume depletion secondary tohyperemesis gravidarum is the most liely cause o$ renal insu$$iciency in this patient with signso$ orthostasis and a bland urinalysis. Other causes o$ acute renal $ailure during pregnancyinclude e>acerbation o$ underlying renal disease! such as lupus nephritis and other glomerulardiseases! and acute tubular necrosis related to sepsis. "reeclampsia is unliely because o$ theearly stage o$ pregnancy and the absence o$ hypertension! edema! and proteinuria.(hrombotic microangiopathy usually develops postpartum or in the peripartum period! but

occasionally occurs earlier in pregnancy. 'owever! thrombotic microangiopathy is unliely inthis patient because o$ the bland urinalysis and the absence o$ signs o$ hemolysis on theperipheral blood smear. Urinary tract obstruction is possible but is less liely than prerenala2otemia. (here is no evidence o$ systemic lupus erythematosus! but this disease can occurduring pregnancy.(he patient should be hospitali2ed $or volume repletion! which is both a therapeutic and adiagnostic maneuver. 3$ the clinical and laboratory $eatures o$ volume depletion and renalinsu$$iciency reverse! there will be no need to consider a diagnosis o$ urinary tract obstruction."lasmapheresis is indicated $or the treatment o$ thrombotic microangiopathy duringpregnancy! but there is no sign o$ this disorder in this patient. (here is no indication $ortherapeutic abortion or administration o$ -glucocorticoids.ibliography.N2erue ;4! 'ewan-+owe 1! Nwawa ;. Acute renal $ailure in pregnancy: a review o$ clinicaloutcomes at an inner-city hospital $rom 68-668. 5 Natl 4ed Assoc. 6696&:8-6&.

#.1on 0"! 1wan 5(! =a$tery 45. =eversible renal $ailure due to the antiphospholipid antibodysyndrome! pre-eclampsia and renal thrombotic microangiopathy. ;lin Nephrol. 66C9:#7-%.%.5ena 4! 4itch FE. =apidly reversible acute renal $ailure $rom ureteral obstruction inpregnancy. Am 5 1idney Dis. 6689 #:C7-8&.

Item 82

Answer: ;0evere e>tracellular volume contraction in a patient with diarrheaEducational Objective: =ecogni2e clinical situations in which the serum creatinineconcentration may be elevated without a change in the glomerular $iltration rate.3n patients with e>tracellular $luid volume contraction! elevations o$ serum creatinineconcentration are due to reductions in the glomerular $iltration rate ?<=@. 3n all o$ the otheroptions listed! the increase in creatinine concentration could be e>plained by mechanismsother than a reduced <=. (rimethoprim and cimetidine cause elevations in serum creatinine

concentration by inhibition o$ tubular secretion o$ creatinine and not by reductions in the <=.1etones in the plasma inter$ere with the 5a$$e reaction $or creatinine determination. A $alselyelevated serum creatinine concentration as a result o$ etonemia can be easily veri$ied by$inding a lower or normal serum creatinine value using the en2ymatic method o$ creatininemeasurement. =habdomyolysis may accompany prolonged sei2ure activity and causesincreased release o$ creatinine $rom injured muscle. 0evere rhabdomyolysis may also beassociated with acute renal $ailure! which is usually due to accompanying hypotension! drugoverdose! or hyperthermia.ibliography.Ducharme 4! 0mythe 4! 0trohs <. Drug-induced alterations in serum creatinineconcentrations. Ann "harmacother. 66%9#7:8##-%%.

Item 83

Answer:   Administer magnesium sul$ate intravenouslyEducational Objective: 1now how to treat hypomagnesemia in a patient with heart disease."atients treated with diuretics may develop hypomagnesemia because o$ increased renal

Page 43: Neurology Answers

8/11/2019 Neurology Answers

http://slidepdf.com/reader/full/neurology-answers 43/43

magnesium losses. 4agnesium depletion may be increased because o$ poor nutrition and adirect renal e$$ect that causes wasting in individuals who consume alcohol. 'ypomagnesemiais associated with hypoalemia and hypocalcemia! which may be re$ractory to electrolyterepletion unless the magnesium de$icit is $irst replaced. (here is no indication to stop thediuretic. ;alcium repletion is liely to be ine$$ective i$ magnesium depletion is causing thehypocalcemia. 4agnesium should be replaced to treat the electrolyte disorders and preventarrhythmias. (here is no indication $or volume repletion in this patient.

ibliography.Elisa$ 4! 4ilionis '! 0iamopoulos 1;. 'ypomagnesemic hypoalemia and hypocalcemia:clinical and laboratory characteristics. 4iner Electrolyte 4etab. 6679#%:&C-#.#.Agus B0. 'ypomagnesemia. 5 Am 0oc Nephrol. 6669 &:88-##. "ub4ed +in